GT Study set #2

Ace your homework & exams now with Quizwiz!

Give a general summary of three main quantitative research designs used in studying giftedness/gifted education as well as other topics. Identify a fourth design that may or may not be considered quantitative.

1. Descriptive Design - seeks to obtain more information about a specific attribute of phenomenon within a field...can be used to defend existing practices, identify problems, develop theories, develop opinions, or identify others' work in the same field....they do not look for casual relationships or manipulate variables. 2. Experimental Design - represent the paradigm for using the scientific method....the main elements of experimental design are randomization, manipulation, and control. Experimenters randomly select participants, i.e. each has an equal chance of selection. Experimenters manipulate some variable(s) in the study, and some participants receive some kind of intervention/treatment. Experimenters also control some variables. A common form of experimental design is the before/after or pre-test/post-test design. 3. Quasi-experimental Design - this type of design is used when control is unfeasible and/or to protect validity when randomization, manipulation, or control is missing. 4. Correlational/ex post facto Design - this type of design looks for relationships (NOT causality) without manipulating variables.

Explain some of the characteristics of a bell curve, the meaning of normal distribution in statistical analysis, and an example of how these terms relate to giftedness and gifted education.

A bell curve, aka a normal curve or Gaussian curve, graphically visualizes a normal distribution. Most values obtained fall near the median (middle/center), and fewer fall higher or lower, resulting in a curve with a bell shape - higher in the center, sloping downward toward the edges, and symmetrical, with equal numbers above and below the middle. With normal distributions, the mean (average of all values), median (the middle of all values) and mode (the value occurring most often) should be equal or nearly so. If a standardized IQ test is administered to a general education class including a few gifted students and the scores are plotted on a graph, the overall distribution of all scores is likely to be normal; however, the few gifted students' scores are likely NOT to be found near the median with most other student scores, but at one extremity, among the fewest and highest values.

Discuss some aspects of education and the IEP for a gifted student that should be considered during an IEP planning meeting, and discuss how parents can advocate for their gifted child by addressing these aspects.

A gifted IEP planning meeting should be student-centered and should be used for developing the IEP, not simply for reviewing a previously written IEP with no input from the meeting. Because school IEP team members will have prepared for the meeting, parents may misinterpret this preparation as the school presenting a predetermined plan. However, all members, including the parents and the students, should contribute to IEP development during the meeting. If a certain class or pull-out program is proposed, for example, but parents know it will not challenge their child (or already does not), they should indicate this. If parents find that the proposed IEP does not meet a certain educational need for the child, they should bring this up for the school to define and the team to find ways to meet it. Parents also can advocate for courses initially not offered by the school or not at their child's grade level.

Generally describe how gifted students are determined to be eligible for gifted educational services in public schools, including how this differs from eligibility determinations for services to disabled students.

A major difference between determining eligibility for special services to disabled students versus special services to gifted students is that federal laws, such as IDEA 2004 and Section 500 of the Americans with Disabilities Act, specifically provide for special services to students with disabilities. The IDEA contains 13 categories of specific disabilities that qualify for special education services. Each covered disability has criteria to meet for a positive diagnosis. Section 500 of the ADA has a general definition of disability, and any student whose condition interferes with education according to that definition is eligible for services. However, the federal laws do not address giftedness as they do disabilities. Each state Department of Education decides how to address giftedness. Most departments begin with their definition of "gifted" to create rules. Local school districts comply with state eligibility rules. Some districts assign each school eligibility team to administer eligibility procedures.

Identify some environmental and person variables that influence the way teachers implement curricula, including those variables based on theoretical models of gifted education.

A number of factors in the environment and within the individual teacher can influence the way teachers implement curriculum units in reading and math. While curriculum units based on theoretical models adhere to principles of the applicable model, their implementation can vary as it is shaped by personal and environmental variables. For example, the grade level of students being taught will make a difference in the way that teachers implement curriculum. The school subject and department also will influence what elements are emphasized or minimized. Even the school building can make a difference in how teachers carry out their curricula. Variables at the school district level also affect teachers' understanding of a curriculum and how they implement it. Teachers' personal variables include their duration of classroom experience, their beliefs about learning and teaching, their particular approach to classroom management, and the grade level(s) and subject(s) they teach.

Define the term "test ceiling" relative to testing gifted students. Give an example of a common test ceiling criterion and its disadvantages.

A test ceiling means a test's upper limit, i.e. the highest score it is capable of yielding. Most intelligence and achievement tests used in schools cannot accurately measure gifted student levels if they exceed the limits the test can measure. For example, many IQ and achievement tests direct administrators to stop when the student gets three items wrong. The disadvantage: a student can get two wrong, one right, one wrong, etc., never meeting the stopping criterion of the three wrong consecutively. The student does miss a number of items, yet never hits the test's ceiling. An additional disadvantage: the student never reaches the point when items become too difficult to answer, which represents the true ceiling. Therefore, that student's score may be accurate, a bit too low, or much too low, but this is impossible to determine. Only certain is that the student's score is his/her lowest possible score.

Define a virtual learning environment (VLE). Give examples of how VLEs can be used in gifted education.

A virtual learning environment is a computer-based environment where learning materials can be delivered to students via the Internet. By using computer information technology, teachers can differentiate their instruction for gifted and talented students. Enhancing school projects by using online databases, scanning images, and incorporating hyperlinks and video clips into them is generally more motivating to students with all levels of ability. VLEs are useful for developing cultural experiences in the visual, performing, and/or creative arts. Students can visit museums, art galleries, government agencies, various industries, and institutions using VLEs. Teachers can acquaint students with a variety of ideas through online exposure to famous and/or controversial individuals. In subjects incorporating research activities, VLEs can give opportunities to gifted students for more advanced learning. Because of the virtually limitless amounts of information accessible on the World-Wide Web and the way it is instantaneously transmitted, using VLEs is a method that students find motivational and exciting.

Explain what ability grouping is in educational settings, and describe its use with gifted students.

Ability grouping should not be equated or confused with cluster grouping, heterogeneous grouping, or tracking. Ability grouping involves dividing the students within one classroom and grouping them according to their ability levels, which can differ broadly within one general education classroom. This grouping by ability is not rigid but fluid, and it can be changed as needed at any time. For example, a student might be placed into a high ability group for math and participate in a group of middle ability for reading. If that student's reading improves, the student could move into the high ability group for reading. Conversely, if a student develops problems in an individual subject, she/he can move into a lower ability group to address his or her academic needs best. The flexibility of ability grouping allows gifted students more options to receive the most appropriate instruction and to adjust for needs that change over time.

Comment on the value to gifted students of integrating the arts with academic subjects and vice versa in their educational programs and activities.

Academically successful gifted students will have gained much experience with using their superior skills of reasoning, abstract thinking, and problem solving in academic subjects such as mathematics, sciences, language arts, and social studies. But when the arts are integrated into these subjects, and reciprocally, elements of academic subjects are integrated into artistic activities, gifted students can apply these advanced cognitive skills in different ways that may be new to them. This type of activity promotes cognitive flexibility and supports the gifted student's diversity of interests. These novel applications of their skills are also facilitated when students experiences what Csikszentmihalyi called "flow", the immersion in the creative process wherein time seems to stop, the artist becomes one with the art, and creation is spontaneous and effortless. With curricular integration of arts and academics, gifted students can contribute more of their unique abilities, insights, and visions, and make original discoveries.

Discuss the issue of acceleration as a form of specially designed instruction accommodation for gifted students in general education classrooms in public schools, including considerations for the student and school district. attitudes vs. policies.

Acceleration is considered a more serious choice than other forms of accommodations for gifted students. Considerations for the student include the student's levels of emotional and social maturity, which can cause adjustment difficulties if a student is placed in chronologically older classes and is emotionally and/or socially unprepared. Historically, students skipped whole grades. While some cases still warrant such practices, students also can accelerate in individual subjects according to their levels of ability and performance. They can even work across grades if their learning levels dictate "straddling" two grade levels. Though accelerating is not always best, research finds gifted students more often benefit from ability-appropriate materials. Objective data on the student's current performance and potential are important to decisions to accelerate. Many school districts frown on acceleration, but they cannot have a policy prohibiting it. Attitudes against acceleration should be challenged when acceleration represents the only way to benefit the student.

Discuss the use of the Wechsler Intelligence Scales for Children, 4th edition (WISC-IV 2003) in identifying gifted students. In addition, identify the subscales included in the test, the composite scores included; and which composite scores are the best and worst indicators of giftedness.

According to a number of educational researchers who have analyzed its content and results, the WISC-IV is a useful diagnostic tool for giftedness. They note that although it seldom gives scores above the 140s and has a score ceiling of 160, it also indicates ability beyond these limits. WISC-IV's ten required subtests include: Similarities, Vocabulary, Comprehension, Matrix Reasoning, Picture Concepts, Block Design, Letter-Number Sequencing, Symbol Search, Digit Span, and Coding. Optional supplementary subtests include: Arithmetic, Word Reasoning, Picture Completion, and Cancellation. The WISC-IV gives four Composite Scores from subscale groups: Verbal Comprehension, Perceptual Reasoning, Working Memory, and Processing Speed. Expert analyses find Verbal Comprehension (a composite of Similarities, Vocabulary, and Comprehension subscales) and Perceptual Reasoning (a composite of Matrix Reasoning, Picture Concepts, and Block Design subscales) very good indications of giftedness, as they assess abstract and visual thinking. Working Memory and Processing Speed are the subscales least correlated with giftedness.

Comment on how instructional strategies are selected in differentiated language arts education for gifted students, and how the diagnostic-prescriptive instructional approach relates to this.

According to expert educators, instructional strategies are not differentiated exclusively for students with gifts in the language arts. Rather the level and character of the particular curriculum being taught is used to determine which instructional strategies are most indicated and how they should be applied. The curriculum and the choice of teaching strategies cannot be separated. Educational researchers and groups such as the National Association for Gifted Children find the diagnostic-prescriptive approach to teaching as of value for the purpose of differentiating language arts instruction for gifted students. This approach affords a process of assessment to determine each individual student's abilities and talents in the language arts, allowing educators to adapt their instruction to meet individual student needs. This lets linguistically gifted students progress at faster paces by not requiring their instruction in skills that they have already mastered.

Describe some features of the Naglieri Nonverbal Ability Test, 2nd edition (NNAT-2) as an instrument for identifying gifted and talented students.

According to the test's author, the NNAT-2 is "culturally neutral," thereby making it suitable for testing culturally diverse student populations. It is nonverbal, making it a good choice for ESL/ELL and nonverbal students. It can be used with ages 5 through 17 years and grades K through 12. It has seven levels corresponding to grades K; 1; 2; 3-4; 5-6; 7-9; and 10-12. It takes 30 minutes and is easy to administer. Students do not need good reading skills, math skills, well-developed vocabularies, or factual knowledge to respond to the test items. For identifying gifted/talented students whose socioeconomic backgrounds have limited their development of verbal skills and acquisition of information, this test is a useful instrument. Among tests of ability, the NNAT-2 has the most recent norms, from 2008. Its administration and instructions are facilitated by pictures. Its graduated difficulty range allows identification of advanced as well as gifted and talented students.

Discuss the general education teacher's involvement with the gifted student's IEP (GIEP), including benefits, GIEP meeting procedures, and parent recourse if teacher refuses to implement a GIEP.

Advocates observe that engaging general education teachers in the GIEP process represents the best way of developing and implementing a GIEP. The general education teacher should attend the GIEP meeting. Since a teacher familiar with the student always should attend IEP meetings, the primary general education teacher also is likely to be on the IEP team and attend the meeting. If a school/district does not include the general education teacher, parents can request the chairperson of the GIEP team or local educational agency (LEA) to include her/him. All teachers must comply with the GIEP. If a teacher refuses to implement the whole GIEP or any part(s) of it, e.g. pre-testing, giving differentiated classwork and/or homework, etc. parents either should call for another Team meeting or pursue due process proceedings. Teachers can make written objections to a GIEP on record. However, team-proposed, parent-approved GIEPs always must be implemented.

Identify some post-reading teaching strategies social studies teachers can use with gifted students to foster critical thinking relative to importance, chronology, true-false statements, and important issues in the text they have read.

After they have read assigned social studies texts, students will benefit if their teachers have them list the most significant points n the material and rank them in order of importance. This will help students clarify priorities among logical points/reasons in an argument, enhancing critical thinking skills. When the time sequence of events is significant, teachers can assign students to list 5-10 chronological events the author has cited. Teachers can help students identify an author's viewpoint by giving them statements to identify as true or false according to the author and having them cite specific pieces of text as the basis for their choices. The teacher also can use true-false statements to help students distinguish authors' opinions from facts. Regarding important issues, teachers can have students evaluate the author's argument, given sufficient time and teacher guidance. This can motivate further reading and research: in the evaluation process, students will want to consult additional sources.

Recount some positive effects of accelerating gifted students that research has found. Give some examples of different forms of acceleration.

Although acceleration must be decided for each individual student, research finds many benefits of acceleration for gifted students. Those students who exhibit advancement for their age far beyond their grade can even develop poor study habits, behavioral problems, become bored and daydream in school, or avoid attending school if they are forced to stay with their age-grade level. Especially in math, science, and English, some gifted students are more gifted than in other subjects. Such students may attend class with higher grades in their most gifted subject(s), but stay with their age's grade level for other subjects. Another form of acceleration is tutoring, either individually or in small groups. Some high school students gifted in math, for example might be in their school for all other subjects, but a group of them would attend advanced math classes with a local university's math professor a couple of days each week.

Describe some factors that contribute to difficulty in measuring creativity.

Although many researchers agree about common elements of creativity such as the originality, applicability, and value to society of creative work, they find it much more difficult to agree about which methods to use to make their definitions operational, and about which instruments of measurement to use. Creativity is such a complex quality that most measures cannot sum it up alone. Experts advise using multiple measures, but even this tactic can fail to achieve a comprehensive representation. Forty years ago, researchers found that the absences of any one unified and broadly accepted theory of creativity caused problems with understanding creativity's relationship to other abilities, the ramifications of different testing instruments and administrations, and operational definitions. Researchers find the same problems today. Meanwhile, so many creativity tests have been developed with varying degrees of psychometric credibility that difficulties in focusing on criteria have only increased.

Explain how environmental influences in the classroom can affect motivation and lead to underachievement in gifted students.

Although one might expect gifted students' exceptional abilities to support them in their classroom performance, research finds that gifted students are also very sensitive to influences they encounter in the classroom environment. While enthusiasm for learning and for subject content is a common characteristic of gifted children, teachers can dampen this enthusiasm when they insist that students conform to traditional classroom procedures and behaviors. Because gifted children often think and do things differently, trying to make them conform can ruin their motivation. They lose interest and get bored. In addition, many gifted children, especially younger ones, frequently do not know how to handle adults' high expectations, how to conduct successful interpersonal interactions, or how to set suitable goals for themselves. All of these factors in combination can contribute to academic underachievement. Underachievement actually is one of the most frequent problems among gifted students.

Discuss some general aspects of integrating learning in the arts into math and science curricula for gifted students, and give examples.

Although people sometimes consider math and science as dealing with facts and objective reality rather than the arts because artists may represent and manipulate reality according to their own perceptions and the messages they want to impart, the arts and sciences actually are closely related. For example, the musical compositions of J.S. Bach follow mathematical formulae and their structures can be analyzed mathematically. Perspective and color are sciences used in drawing and painting, as are anatomy and physiology used in portraiture. Famous Renaissance polymath Leonardo da Vinci not only did work far ahead of his time in both the arts and the sciences, he also integrated these fields in everything he produced. Contemporary artists Thomas Locker has created portfolio formats specially designed for teachers to use in classrooms. The combine his beautiful paintings of nature subjects with information and activities that teachers can use to foster scientific inquiry by gifted students.

Identify an important general purpose of program evaluation regarding the relationship of program planning and implementation in gifted education. List a number of aspects that educators can examine to fulfill this purpose.

An important reason to evaluate gifted programs is to ascertain whether the school's/educators' implementation of their program is congruent with their initial program plans and/or to what degree. To determine this, they can examine: Identification of gifted/talented students, services provided to them, data available showing how effective their screening/identification systems are, criteria for screening and identification to prevent overlooking special populations, available curriculum options for meeting student academic needs, data available showing how effective the curriculum is, whether/how acceleration is used, how effective their acceleration options are, how program goals and objectives are implemented, how these are connected to district philosophy and mission statements, impacts of the program on the regular education program(s), opportunities available for advanced training to all teachers, how formal and informal feedback is used for program quality improvement, student educational outcomes, curriculum approach implementation at different grade levels, and evidence showing the value of current service deliver models.

Discuss in general some current challenges general education teachers face with gifted and talented students in their classrooms and have all information technology can help to address the challenges.

And ever increasing challenge in our schools is that due to economic considerations, school budgets are being cut, and special or supplementary services to get to students are often eliminated. As a result, do you know education teachers fun they have more gifted and talented students in their classrooms. Their challenge involves meeting the educational needs of students with average ability, it is what hi there lady, just with disabilities who is inclusion in regular classrooms as mandated by law, and students with both gifts and disabilities. Teaching the same curriculum to all students, I need to teacher centered instruction, is no longer feasible. Information-technology recently has become popular for use with gifted students. As such, information-technology can replace some existing delivery methods and be added to others to enhance them. Is it a technology also aids students with vision, hearing, physical, and other disabilities. Those students with both guest and his abilities receive multiple benefits from technology. Students can work at their own pace is, access multiple modalities, select preferred modalities, and learn in more depth through enriched learning experiences.

Describe some factors that parents of gifted children should consider in judging the quality of a gifted program or school.

Any good gifted education program or school should state its learning outcome expectations clearly for the student. Providing activities that students enjoy is important, but gifted students also must learn while having fun. Specifying learning objectives and designing instruction to facilitate reaching those objectives is an essential part of any gifted program. Gifted education programs/schools should provide curricula that stimulate and challenge their students. Gifted students often grow bored if instructional tasks are too easy and do not require them to "stretch" their minds. Flexibility is important to help gifted students receive suitable challenges and meet individual needs. For example, students who perform several years above grade level in one subject should not be made to do grade-level work. Programs should be flexible to let such students study higher-level material. Students with musical gifts could exchange some school time for participation in special music programs, or study with an "artist-teacher" musician.

Give some examples of learning goals and activities whereby teachers of gifted students can incorporate arts instruction into language arts instruction in the area of reading.

Arts instruction reinforces spoken and written communication and gives gifted students more opportunities to use analytical thought and creative problem-solving skills. Having students visually illustrate (draw, paint, etc.) a salient aspect of a story they read and discuss these illustrated aspects in small groups enhances the students' critical thinking skills. Teachers can help students adapt a written story into a dramatic piece, choose the most important scenes, explain their selections, and perform it as actors and narrators. This type of activity stimulates not only analytical thinking, but also imaginative interpretation and reading. Teachers also can have gifted students identify a problem, conflict, or issue in their assigned reading; assign the parts of characters included in the text to different students; and then debate the topic they have identified. This activity can make gifted students more aware of various points of view and of people's different motivations for their behaviors.

Give some examples of aspects within the areas of content and process that teachers should consider when selecting curriculum materials for their gifted students.

As Renzulli (2000) defines the,. content equals authentic knowledge and process equals teaching techniques. Within the area of content, when selecting curriculum materials, teachers should consider: the organization of the content, the depth in which the content covers its subject, the accuracy of the content, and the degree to which the content moves from concrete topics to abstract concepts. Within the area of process, teachers should consider: opportunities to use/learn problem-solving skills, opportunities to use/develop communication skills, activities that exercise skills of reasoning, relevant connections among parts of the subject area, providing multiple representations of concepts to be taught, opportunities for students to verify their speculations when indicated, opportunities for students to work both individually and in groups, and opportunities for students to use divergent thinking in responding to a lesson, exercise, or activity.

Characterize the attitude of many experts in gifted education toward the affective development of gifted students. Describe a common position of organizations supporting gifted students regarding affective development.

As many experts in gifted education assert, gifted students deserve for adults to attend not only to their gifts, talents, and performance, but also to their overall wellbeing. Gifted children go through developmental processes and achievements universal to all children, although they may experience these processes and achievements differently. They also have other developmental experiences unique to each individual. Because educators and other adults so frequently focus on the academic and creative potential of gifted students and their performance or lack thereof, they may not focus enough on the affective dimension of their development. Adults need to remember that gifted students' developmental progress is not only cognitive, but also emotional, social, and career-related. Moreover, progress in cognitive, but and academic areas also has emotional and social effects. Thus, organizations such as the National Association for Gifted Children take the position that teachers, administrators, and counselors in schools should deliberately, proactively nurture the emotional and social development of gifted students.

Explain three ways that educators can use the Wechsler Intelligence Scales for Children, 4th edition (WISC-IV, 2003) as a more accurate index of giftedness that using its Full-Scale IQ (FSIQ) score, including using the Dumont-WIllis Index-1 (DWI-1), Flanagan and Kaufman's General Ability Index (GAI, 2004), and a simple practical approach to identification, service provision, and accommodations.

At the Gifted Development Center, Silverman, Gilman, and Falk (2004) found that gifted students' WISC-IV Composite Scores in Verbal Comprehension and Perceptual Reasoning were typically high enough to qualify them for gifted educational services, but their scores on Working Memory and Processing Speed subscales were typically below qualifying levels. Additionally, since the Full-Scale IQ (FSIQ) score average incorporates these lower scores, they drag the average down, and also are individually not good indicators of giftedness. Consequently, the FSIQ identifies neither gifted strengths nor relative weaknesses. a. The DWI-1 computes a score combining the Verbal Comprehension and Perceptual Reasoning Composites using only six WISC-IV subscales, which these researchers find and excellent option for schools to identify gifted students. b. Flanagan and Kaufman's GAI averages the same two Composite scores; Harcourt Assessments (PsychCorp) trainers support this. c. Another solution involves basing identification on assessments of reasoning, delivering gifted services/accommodations, and adding accommodations for relative weaknesses.

Discuss some of the prevalent social characteristics of intellectually gifted students.

Because intellectually gifted students are proficient in understanding abstract concepts, they often show an interest in social and philosophical issues. They are typically concerned about seeing justice done and tend to object to actions or decisions they perceive as unfair. Developmental psychologists such as Jean Piaget and Lawrence Kohlberg would say that such children display advanced moral reasoning. Because they have high standards for themselves and others as well as the ability to do many things better than others, even achieving perfection in certain tasks, they tend to be perfectionists. They often have more internal motivation than external motivation to achieve. Desires such as wanting good grades and teacher approval are external, whereas desires such as wanting to learn about a subject or to beat one's own personal best on tests are internal. Other prevalent characteristics include: good relational skills with parents, teachers, and age peers; well-developed senses of humor; energetic approaches; and emotional and physical sensitivities.

Discuss some of the criticisms or disadvantages, as well as some of the benefits or advantages, of qualitative research as opposed to quantitative research in gifted education.

Because quantitative forms of research using statistical analysis are the predominant methods used in educational and psychological research as well as in other social sciences, critics of qualitative types of research often regard these methods as more subjective. However, as Myers (2002) states, "Since we maintain our humanity throughout the research process, it is largely impossible to escape the subjective experience..." This humanity includes intuitive realizations and "aha moments." Qualitative research demands extensive time and effort. Furthermore, because of the subjective nature of qualitative research result, these results obtained with samples cannot be generalized to larger populations in the same manner as can the results from statistical studies. However, Myers and other researchers (e.g. Neill, 2006) find qualitative methods to transcend those drawbacks with other redeeming qualities, such as in-depth descriptions with sufficient detail for readers to realize the idiosyncrasies of the individual research situation, and providing the researcher's perspective on that situation.

Discuss the use of the phrase "above grade level" to describe a gifted student's school performance relative to developing the gifted IEP.

By itself, the phrase "above grade level" contributes no meaning to establishing a gifted student'sPresent Levels of Educational Performance (PLEP) or to developing that student's IEP. Some gifted advocates opine this phrase should not be included in gifted IEPs. If it does appear, both the student's current grade level in school and the grade level at which the student performs must be defined. In some U.S. states, State Departments of Education require these determinations by law. Objective, standardized achievement tests have established norms for each grade level, so these definitions can be determined through test scores and norms. The description "above grade level" without such specific grade-level information is analogous to going clothes shopping for a child with only the information that she/he is "taller than four feet." The child's actual height must be measured; so must the student's actual performance grade level be measured, as well.

Identify some areas addressed in theoretical models of gifted instruction and in quantitative and qualitative research into gifted education. Describe some research questions in these areas.

By studying theories of gifted education, researchers study theoretical models to address areas that include intelligence, identification, evaluation, alternative assessment, curriculum, programming, and professional development. Some common research questions among studies include these. Researchers want to know whether students from under-represented groups such as students with identified disabilities and abilities, students from economically disadvantaged populations, and Blacks and Latinos, will be identified in greater numbers by expanded criteria than by traditional criteria for identification. Researchers want to discover whether students identified as gifted by both traditional and non-traditional methods perform better on standardized achievement tests, extended standard-based assessments, and/or structured performance assessments when given the model-based curricula, than such students given their schools' general education curricula. Additionally, researchers want to know if students identified using traditional criteria outperform students identified by expanded criteria on standardized and performance-based measures.

Explain one differentiation between the definitions of giftedness and talent.

Canadian psychology professor/researcher Dr. Francoys Gagne (not to be confused with American educational psychologist Dr. Robert M. Gagne), in his "A Differentiated Model of Giftedness and Talent (DMGT)" (1985, updated 2000), clearly distinguishes giftedness from talent using the criteria of competence/ability/aptitude versus performance/skill/achievement. He finds that "...superior natural abilities (called aptitudes or gifts), in at least one ability domain..." that constitute giftedness are "untrained and spontaneously expressed." He defines talent as "...superior mastery of systematically developed abilities (or skills) and knowledge in at least one field of human activity..." The differences are 1. that talent requires methodical development and learning, while giftedness needs no training; and 2. that gifts are in "ability domains" while talents are in "fields of human activity." He designates both as placing an individual in the top 10% of age peers in a given domain or field. His ability/aptitude domains include Intellectual, Creative, Socioaffective, and Sensorimotor.

Describe some of the characteristic of the profile of an "underground" gifted student.

Commonly researchers have found that middle school females often make up a group of students classified as the "underground" type of gifted student. In pre-adolescence and early adolescence, girls' need for group belonging and peer acceptance increases exponentially; as a result, many girls will hide their gifts or deny them to fit into a group. The researchers note that gifted boys in this category tend to go underground later, typically in high school, as a reaction to the pressure to join athletic teams and events and to their desire for identification with more popular "jocks" rather than less popular "nerds". Anxious and insecure, "underground" students have needs that conflict with adult expectations. Adult pressure only worsens their denial and resistance. Experts find that they benefit more from acceptance. As such, experts advise adults neither to push nor to permit abandonment of all prior interests in order to fit in; instead, adults should help these students find alternatives to meet their academic needs during what is considered a period of transition.

Identify what can be accomplished by curriculum compacting for gifted students, when it is applied, and steps in the compacting process.

Compacting is a way to streamline regular curricula for gifted students who learn faster. Compacting also can fill instruction gaps for students accelerating to higher grades and for those using the time saved by compacting to pursue enrichment activities. Sequential steps in the curriculum compacting process include: identifying pertinent learning objectives for the subject or grade level; locating or creating pre-test instruments for these objectives; identifying student candidates for compacting; pre-testing these candidates to ascertain their learning levels of the identified objectives; get rid of teaching, drilling, and/or practicing time for students who have already mastered the objectives; for objectives students have not mastered, streamline their instruction for the gifted students' faster rates of mastery; offer options of enrichment of acceleration for students who have exhausted the regular curriculum; and maintain records of the entire process and of instruction options available for students participating in compacted curricula.

Define content skills versus process skills in gifted education, and provide some examples. Explain the relationship between content skills and process skills.

Content skills involve acquiring and retaining the information presented in a specific instructional subject area. Examples include learning the correct spelling of words; knowing the correct meaning of vocabulary words; knowing factual material such as places, names, and dates in history or social studies; remembering mathematical equations or formulas, etc. In contrast, process skills involve more interpretation of information. For example, a students not only knows what reportedly happened in a historical event, but she/he also can interpret that event in terms of its historical context, such as the frame of reference and viewpoints affecting participants. Process skills include analyzing information through comparisons-contrasts; generalizations; finding cause-and-effect relationships; classifying and sequencing; summarizing; and drawing inferences, conclusions, and predictions. Such skills include applying critical thinking skills and systematic methods of inquiry, especially in sciences. Content and process skills are related because content must be learned to process it. Integrating the two is most effective.

Explain the meanings of and difference between convergent thinking and divergent thinking relative to the study of creativity, and provide examples.

Convergent and divergent are opposites. In convergent thinking different thought narrow/come together (converge) on one conclusion. An example is forensic detective work. If a suspect's fingerprints are found on the murder weapon, his DNA is found on the victim, and eyewitnesses place him at the scene of the crime at the time of death, then these pieces of evidence converge to show that suspect's guild. Considering whether this suspect was framed or whether the evidence was circumstantial would involve some divergent thinking in which different thoughts proliferate/separate (diverge) into many possibilities. Some creativity tests ask takers to generate as many different uses as possible for a brick, tin can, paper clip, etc. Listing all the things you can build with a brick is not considered an instance of divergent thinking, because building is one category. Holding papers/cloths/leaves together with a paper clips is convergent; using paper clips as snowshoes for mice is divergent.

Describe the category of correlational research in terms of how gifted students can use it in independent study projects, including examples.

Correlational research is a type of quantitative research method that compares quantitative data obtained to discover whether a relationship exists between two or more variables. Specifically, it seeks to find whether one variable affects another variable and how. A positive correlation means that as one variable increases or decreases in amount or number, so does the other variable. A negative or inverse correlation means that as one variable increases, the other decreases, and vice versa. Statistics also show the strength of the correlation, or the degree of relationship. Some example include whether the seasons affect school attendance, whether an individual's head size and foot size are related, or whether school attendance affects achievement test scores. Correlations do not imply causations: two variables may increase or decrease together or oppositely, but this does not mean that either variable causes the other. To determine causation, experimental research must be used.

summarize some common difficulties in writing curricula for gifted students. Summarize the rationale and differentiating characteristics of one curriculum development model.

Curriculum writing can be quite difficult and time-consuming. Difficulties include trying to comply with state curriculum guidelines, feeling pressured to include activities currently popular in the educational community, trying to achieve a balance between content (information/knowledge) and process (teaching/learning), and juggling all these demands. Moreover, the results of these efforts often do not further education or impart meaning to it. Seeking to improve curriculum writing and believing that teachers need time and tools to create meaningful teaching units based on desired outcomes named in a curriculum guide, Renzulli et al (2000) of the University of Connecticut's National Research Center on the Gifted and Talented produced the Multiple Menu Model of curriculum development. Differentiating this model for more traditional approaches are its stronger emphases on encouraging original inquiry by students, on examining the organization and interrelatedness of knowledge, and on arriving at a balance between content and process.

Discuss some of the characteristics of imagination in gifted children.

Dabrowski identified the following sensitivities in gifted children: psychomotor, sensual emotional, intellectual, and imaginational. Many gifted children have very vivid imaginations. They are likely to have vivid dreams and to recall them. Because their imaginations allow them to conceive of diverse possibilities, to see things from different viewpoints, to look at things in new/different ways, to be inventive, and to generate original and unusual ideas, these children are likely to develop and utilize good senses of humor. They also are likely to daydream and have imaginary friends. They tend to love fantasy, poetry, drama, and music. They are more likely to have strong visualization skills as well as mental images with more detail than the mental images of other children. Because they can imagine all sorts of possibilities, they can envision the worst case scenarios for any situation. This characteristic can promote fear of the unknown, of takings risks, and of new situations.

Identify five domains of ability or aptitude wherein a student may be gifted and explain how a student might demonstrate giftedness in each domain.

Domains of aptitude (identified by F. Gagne) are 1. Intellectual - A student who is gifted in the Intellectual domain might demonstrate this giftedness through superior learning, academic and/or otherwise, as evidenced by extent, breadth, depth of knowledge and/or speed and ease of learning; through superior achievement in school grades and standardized tests; and/or facility beyond grade level in reading or mathematics, etc. 2. Creative - might be demonstrated by advanced learning and understanding of principles in music, art, etc.; superior musical interpretation, performance, and/or composition; superior composition and execution in visual arts; originating mathematical/scientific formulas; or producing inventions. 3. Socioaffective - might appear in a students' heightened ability in making friends, helping peers resolve conflicts, disarming bullies/hostility, and empathy for others. 4. Sensorimotor - can emerge in athletic abilities or physical abilities for certain crafts and arts. 5. Others - includes extra-sensory perceptions such as precognition, clairvoyance, and communicating with spiritual realms.

Describe the three component characteristics necessary in order for a student to demonstrate "gifted behavior".

Dr. Joseph Renzulli and colleague Dr. Sally Reis, both from the University of Connecticut, created a Schoolwide Enrichment Model (SEM) (1976, 1977, 1985, 1997) for academically gifted and talented students and for all schools to develop students' strengths and talents. Renzulli focuses more, as do other behaviorists, on students' actions-i.e. gifted behavior-than on students' abilities/potentials. He proposes that three groups of characteristics must interact to produce gifted behavior: 1. above average abilities, either general or specific 2. high levels of motivation 3. high level of creativity. He identifies any child who succeeds either in having or in developing this combination of characteristics and applying it to any "potentially valuable" field of human activity as gifted or talented. Renzulli states in the SEM that gifted behaviors exist "in certain people (not all people), at certain times (not all the time), and under certain circumstances)."

Describe some typical procedural safeguards for American state Departments of Education regarding the education of gifted students.

Each state Department of Education writes its own procedural safeguards. Specifically, many states require that if parents obtain independent evaluations of their gifted children at their own private expense, the state's school districts must consider those evaluation finding in any school decisions they make for those students. In addition, states commonly require that if an administrative law judge requests an independent evaluation as part of a hearing, the expense becomes a public responsibility. Usually the district must finance the evaluation. Another standard of state procedural safeguards, adopted rom Federal disability laws, is the right of gifted students and their parents to a due process hearing regarding the student's identification, evaluation, or educational placement. States also commonly stipulate that students remain in their current placements while and administrative or judicial proceeding is pending.

Explain the relationship of learning in the arts to the cognitive and academic development of gifted students and the implications of this kind of learning for instructional design.

Educational research has found that in addition to developing gifted students' creative abilities, instruction in the arts also enhances their cognitive and academic accomplishments. Art education improves the student's observational skills, abstract thinking skills, analytical skills, and problem-solving skills. In creating works of art, artists must use their reasoning powers to identify and define problems, to visualize and establish their goals, select methods for gathering information, propose solutions to problems, and evaluate their solutions, and use their imaginations to revise those solutions. Therefore, the creative process in the arts requires, develops, and exercises higher-level cognitive skills. Teachers can integrate the arts into their curriculum and design activities tailored to gifted students' unique abilities, needs, and interests, and challenge them with increasing sophistication and complexity in the activities. Educational researchers advise teachers designing parallel curricula or differentiated instruction to clarify their students' educational goals before developing alternative instruction. Arts learning goals and activities can be integrated into math, science, language arts, and social studies curricula.

Discuss how our schools and educators can foster the creative performance of gifted students.

Educational researchers find that modifying curriculum, using alternative materials, encouraging brainstorming, and teaching strategies for thinking "outside the box" are all necessary, these strategies alone are not sufficient. Teachers, students, parents, and administrators need to collaborate to affect changes in the entire climate of our schools at every level, as well as in the environments of individual classrooms. Some gifted education programs developed in recent decades can be individualized to meet the needs and interests of each specific student. Instead of identifying only students who excel in traditional school subjects, programs taking this alternate approach acknowledge student talents and strengths across a broader range of dimensions. Researchers find the motivation of gifted/talented students is especially susceptible to classroom environmental influences, even more so among the culturally diverse and /or economically deprived. Experts call for educational reform that does not require money so much as commitment to change and willingness to collaborate to make school environments nurture internal motivation and creativity.

Describe some of the characteristics of an "autonomous learner" type of gifted student.

Educational researchers find that parents may observe "autonomous learner" types of gifted children exhibiting signs of this style at home, but these children typically demonstrate it somewhat later in school once they have learned the system. Unlike the "successful" type of gifted student, who seems to have high self-esteem but depends on adult direction and reinforcement, autonomous learners have strong internalized self-concepts. Rather than meeting the system's demands with as little work as possible as do "successful" gifted students, autonomous gifted learners employ the system to create new opportunities to meet their own needs. Self-directed and independent, these students are characterized by self-acceptance and risk-taking ability. They feel secure about forming personal and educational goals. They exhibit a strong sense of self-efficacy (belief in their ability to do things) and personal power. They are active rather than passive, and they express their needs, emotions, and goals with freedom, but also within appropriate limits.

Give some examples of how differentiated instructional strategies can use techniques matched with advanced curricula to promote effective teaching of gifted students in the language arts.

Educational researchers have found that certain instructional strategies can be paired with advanced curricula for effectively teaching gifted students in the language arts. One such strategy is questioning: when students have read or viewed challenging materials, teachers can stimulate class/group discussions at higher cognitive levels by asking them questions about the material. Another technique teachers can use is to assign open-ended activities. As long as these activities are sufficiently difficult, they can promote learning and growth in gifted students. Another strategy teachers can apply in conjunction with an advanced curriculum found especially effective for gifted students involves giving them poorly structured problems, which facilitates problem-based learning and challenges advanced learners to exercise and develop their problem-solving skills while applying the information and concepts that they have learned at the same time.

Explain some aspects of uneven development in intellectually gifted children and provide examples.

Educational researchers have found that intellectually gifted children do not always develop evenly across all domains of ability. While some gifted students excel in all areas, particularly in the higher grades, many more of these students demonstrate comparatively greater gifts in certain domains than in others. One example of uneven development very common among younger gifted children occurs when their cognitive and conceptual abilities develop far in advance of their physical motor skills. When this uneven development happens, while a young child is capable of more sophisticated ideation, he or she is incapable of executing his or her ideas physically due to less advanced motor abilities. This inequity proves frustrating and produces tantrums in many children. Another example of uneven development occurs when an intellectually gifted child uses his or her superior cognitive skills to interact with the environment almost exclusively, leaving his or her emotional and social skills underdeveloped from lack of practice.

Describe some salient characteristics that research has found in low-income black parents whose gifted children were underachievers in school.

Educational researchers have found that while low income black gifted students who achieved well in school had parents with positive optimistic attitude and high expectations for their children success, those students who underachieved academically had parents who did not feel as optimistic, expressing attitudes of helplessness and hopelessness. Parents of underachieving students in this population were less assertive about involvement in their children's educations then the parents of high achieving students in this population. As a result they were not as engaged in their children's schooling. Parents of these underachieving gifted black students were more likely to have unrealistic and/or unclear or ill-defined expectations for their children. These low income black parents of gifted students to underachieved in school also were found to have less self-confidence in their parenting skills and parents of higher achieving students in this population.

Identify a need in to program management of gifted education programs, as well as three signs of progress in gifted programs that educational researchers have found.

Educational researchers have observed that in some gifted programs, little time and few resources are left for the crucial components of curriculum design, program evaluation, and staff development, because so much time and effort are disproportionately devoted to the identification of gifted students. Nonetheless, researchers have clarified that this observation reflects not a criticism of program managers but instead an insight about the unrealistic expectations and insufficient investment in quality programs for students with high ability. Because trends increasingly emphasize curriculum planning and student assessment for such quality programs, greater program management capacities are needed. Progress includes greater recognition and provision of a variety of service options at the program level to meet diverse student and community needs, program developers' responses to the National Association for Gifted Children's standards for conveying program expectations, and the positive perceptions by constituents of the quality of staff in good gifted programs.

Summarize some areas of the affective development of gifted individuals wherein further research is needed according to experts in the field.

Educational researchers specializing in gifted education find that many aspects of the social and emotional development of gifted individuals have not been researched nearly as much as the intellectual and academic aspects of their development. They recommend that researchers, school psychologists, guidance counselors, and other professionals can contribute to exploration of the affective domain. Asynchronous development in cognitive vs. affective domains has long been observed in the gifted, but it demands further study, as does perfectionism in the gifted. Scientists also want more research into underachievement, depression, eating disorders, self-injurious behaviors, and substances abuse relative to giftedness, as well as responses to life events such as divorce, loss and grief, serious illness, injury, and relocating. Additionally, research into giftedness is lacking related to obsessive-compulsive disorder, sexual abuse, physical disabilities, Asperger's syndrome, serious conflicts between parents and their gifted children, and problematic developmental transitions.

Define cognitive process models used in gifted education. Identify one of the most famous models used. Name and define the parts of this model.

Educational scientists develop theoretical models and then apply the models to instructional approaches and methods. Some models focus on a particular domain, such as affective or cognitive. Cognitive process models focus on the cognitive domain, i.e. mental processes by which students learn and think. Bloom's Taxonomy, created by Benjamin Bloom in 1956, remains one of the most famous and popularly used cognitive process models. This taxonomy contains six ascending levels of cognitive processes: 1. Knowledge: Students can recall or recognize specific information 2. Comprehension: Students not only remember, but understand information they learn 3. Application: Students can learn ideas, principles, and theories and then apply them to other contexts 4. Analysis: Students can break information down into component parts 5. Synthesis: Students can take constituent parts and think originally and creatively to combine them into a whole 6. Evaluation: Students develops standards and criteria and apply these to concepts, methods, and materials presented to judge their value

Give some examples of questions that teachers of social studies can ask gifted students to facilitate their exploration of multiple perspectives.

Educators advise social studies teachers against adhering to a single source or a narrow definition of sequences of historical or current events. Instead, they recommend encouraging student appreciation of a breadth of perspectives within the subject's topics. Teachers can do this by asking students to consider topics such as: from whose viewpoint an account was written; whether other views or interpretations might exist, and if so, why; whose voices are identified in the account and whose voices are not; what evidence is given in the account for its assertions; how the students can evaluate the quality of that evidence; how specific individuals and/or groups of people are depicted in the account; why they might be depicted in that way; why different accounts of historical events exists; and what influence the existence of varying accounts has on our concepts of historical accuracy and of "truth".

Discuss some aspects of defining the concept of giftedness and the manner in which the National Association for Gifted Children defines giftedness.

Educators and advocates agree that no universal consensus exists regarding the definition of giftedness. The National Association for Gifted Children (NAGC) assets that the concept of giftedness, like the concepts of intelligence and talent, is fluid in nature. Depending upon culture and other contexts, such concepts can be manifested and perceived differently. Moreover, the term "gifted" has a variety of meaning and subtleties within each meaning, even in school systems and within individual schools. In any given school, individual educators likely hold a range of different personal beliefs regarding this term. The NAGC finds that individuals showing "...aptitude (...exceptional ability to reason and learn) or competence (documented performance or achievement in to 10% or rarer) in one or more domains" (2008) are gifted. They defind domains as structured activity areas within symbol systems such as language, math, or music, and/or within sensorimotor skill sets such as art, dance, or sports.

Describe some supportive strategies educators can use to promote achievement and/or reverse underachievement in minority gifted students.

Educators should give minority gifted students opportunities to talk about their concerns with teachers and counselors. In addition, educators should employ more collaborative and cooperative group learning methods, as well as verbal and other positive reinforcements. They should reduce norm-referenced and competitive contexts, set high expectations for minority gifted students, and teach for mastery. They should try different teaching modalities such as concrete/abstract, visual/auditory, etc. to identify students't learning styles, strengths, and needs. They should accommodate their instruction to student learning styles. Educators should attend to issues student may have with self-perceptions, self-efficacy (self-perceived ability to accomplish tasks), and motivation. Classrooms should be student-centered and address affective needs. Teaching and counseling techniques should be multicultural. Educators also should enlist significant involvement by students' parents and other family members and engage role models and mentors for their minority gifted students.

Discuss the importance of program evaluation in gifted education, including definitions of formative and summative assessments, and a common error that educators may make in evaluating new programs.

Evaluation plans are crucial elements in the provision of services and programs for gifted students. Formative evaluations are made during the implementation of a program for educators to see that their programs accomplish their intended goals. Summative evaluations are made following full program implementation to assess the extent to which the program attains its goals and objectives. To enable both formative and summative assessment of a gifted education program, the element of evaluation should be built into the original program plan. A common error made in some programs involves waiting to plan the evaluation until the program has been in use for a year or two. Educators may wait this period in order to ensure their program is fully implemented prior to assessment, but planning should be done initially to allow both formative and summative assessments and to assure that the services offered can be evaluated in both these ways.

Describe some characteristics of student populations likely to be under-represented in gifted education programs, contributing factors, and social and emotional effects.

Experts in the field of gifted education have emphasized the necessity of using multiple assessment instruments and criteria for determining student gifts requiring special educational programming. However, too often the need for additional assessments is determined by an initial standardized test score. Such tests can overlook many gifted students. Factors interfering with student performance on standardized tests and/or in the classroom include: difficult life circumstances, lack of parental support, minority cultural values and behaviors, lack of language proficiency, disabilities, depression, lack of faith or trust in schools, behavioral issues, and illness. When such factors prevent identification, gifted students lose opportunities to have their abilities acknowledged and cultivated and opportunities for social access with their intellectual peers. Even identified gifted students frequently have to adjust themselves to a program, instead of the program's adjustment to accommodate students' abilities, as should occur. Without individualization, "one-size-fits-all" programs can cause frustration and discomfort to immigrant, minority, low-income, and underachieving gifted students.

Discuss the status of legal issues concerning gifted students, including federal and state laws, and identify a government agency and a national nonprofit organization that address such issues.

Federal law protect the legal rights of students with disabilities but not the legal rights of gifted students without disabilities. Even the Jacob K. Javits Gifted and Talented Student Act (1994), which funds research and projects related to giftedness, does not protect gifted students' legal rights. Each U.S. state has jurisdiction over its educational procedures. However, discrimination based on giftedness is equally as unlawful as discrimination based on race, color, national origin, gender, or disability. Despite the lack of specific federal mandates for gifted education, the U.S. State Department of Education's Office of Civil Rights protects the education rights of gifted students participating in government-funded programs and activities. From 1985-1995, this office has ruled in 86 cases involving gifted students regarding discrimination, identification, and program admissions. The American Civil Liberties Union has also filed discrimination suits; for example, against a school district for offering fewer Advanced Placement programs in schools with African-American and Hispanic populations with lower socioeconomic status.

Give some examples of mathematical word problems that afford increasing levels of difficulty and complexity that a teacher can give math-gifted elementary students to allow challenges appropriate to different ability levels.

First level: A gas tank was 1/4 full. You add eight gallons and it is now 3/4 full. How many gallons doe this tank hold? Second Level: Sound travels one mile in 5 seconds. You yell near a rock wall and hear the echo 20 seconds later. How far are you from the wall? Third Level: A sprinkler throws water in a circle 15 feet away. It is in the center of a lawn 30x30 feet. How many square feet does the sprinkler miss? Highest Level: Teachers can present class awards to students solving these problems for positive reinforcement, teaching them to appreciate their gifts, building self-esteem, and enhancing motivation*. Alice, Barbara, Charlotte, and Deborah are sisters inheriting money from an aunt, Alice got 1/2 the money. Barbara got 1/4. Charlotte got 1/5. Deborah got the rest, which equaled $1,750.00. What total amount did all four sisters receive?

Define the terms fluency, flexibility, originality, and elaboration in terms of the student of creativity, and state one place these were used in a testing instrument.

Fluency, flexibility, originality, and elaboration are four qualities considered by many experts as key components of creativity. The original version of Torrance's creativity scales measured these four qualities. Fluency - refers to an individual's ability to generate a large number of concepts or of alternate solutions to a problem. Flexibility - refers to an individual's ability to approach a problem from a variety of perspectives, therefore enabling the individual to come up with ideas and solutions to problems in a variety of categories. Originality - refers to an individual's ability to generate unique or uncommon ideas. Elaboration - refers to an individuals ability to add details to embellish one's ideas or results in creative thinking. Fluency, flexibility, and originality are hallmarks of creative thinking. These qualities also are reflected in divergent thinking, a key characteristic of creativity involving generating many different possibilities rather than narrowing ideas down to one conclusion, as in convergent thinking. Elaboration in creative thinking involves the ability to add details to embellish one's ideas or results.

Discuss some general considerations for educators and parents about conducting the Gifted IEP (GIEP) meeting.

For a GIEP meeting, educators typically will prepare some provisional educational goals, courses, and activities they deem appropriate for the student. Furthermore, educators should avoid using technical jargon so parents can understand it. Parents should bring their ideas to the meeting. They can inform educators of their child's interests, likes and dislikes, and behavior, and bring suggestions for educational programs and/or materials they want for their child. They also should ask questions about anything they do not understand or if they need more information. When educators present parents at the meeting's end with a document to sign, often called a Notice of Recommended Assignment (NORA), parents should not assume they must sign it immediately. State laws allow a number of days for them to decide. Parents can request additional and/or ammeded wording. They can also write exceptions to parts they do not accept, and then sign, or they can reject the entire GIEP.

Describe an example of a school's single-subject acceleration plan in science for a gifted elementary school student.

For a fifth-grader accelerating to sixth-grade science, the plan can provide: The fifth-grade teacher and/or gifted intervention specialist will find/create curriculum that compacts assessments on fifth-grade science standards unfamiliar to the student, which she completes in free time at school and/or home with parental help. The staff person assigned as monitor meets weekly with the sixth-grade teacher to discuss student progress and weekly with the student to discuss her adjustment throughout the transition. The student uses the elementary school's webcam/Skype/similar software for virtual attendance to sixth-grade science class at the middle school. The technology coordinator gives student and teachers a short tutorial to facilitate this. After transition, to assure ongoing progress, dual-credit courses in 7th and 8th grade are arranged so after one high-school semester, the student becomes a sophomore eligible to take state graduation tests that spring. High school credit is awarded for completing 9th-grade science in 8th grade.

Describe an example of the transition phase of a school's single-subject acceleration plan in math for a gifted elementary school student.

For a first grader able to learn third-grade math, the school's single subject acceleration plan could include the following provisions: The first- and third-grade teachers will schedule math at the same times. The third-grade teacher is the teacher of record for math. The student is not required to do any first-grade math assignments. A school staff member assigned to monitor the plan meets weekly with the student to assess her success and satisfaction and weekly with the third-grade teacher to discuss any areas of difficulty for the student. Throughout the transitional period, the student's parents are lent copies of the first-, second-, and third-grade math textbooks so they can help her at home with studying and homework. Until she can make new friends in the third-grade class, the student is assigned to sit next to a student in the third-grade class who is her friend and her next-door neighbor at home.

Describe an example of the section of a school's single-subject acceleration plan in math for a gifted elementary school student that addresses assuring continuing progress after the transition phase.

For a gifted first-grader placed in a third-grade class for math who has successfully completed the transition phase, the section of the school's plan to support her continuing progress could include provisions for the following strategies: In fourth grade, arrange for her to take an online math course at the sixth--grade level, and another at the seventh-grade level when she is in fifth grade. The plan can incorporate an option for her to complete an Introductory Algebra course when she is in the seventh grade, or sooner if she is ready for it. Satisfactorily completing the online courses and Algebra/other educational options in the plan will be treated as the equivalents of completing traditional courses as prerequisites for enrollment in advanced high school-level math courses. The plan can provide that the student receives high school credits for completing Algebra and other high school math courses wile she is in middle school.

Compare and contrast some advantages and disadvantages of using qualitative and quantitative research methodologies in measuring giftedness.

For educators who want to determine the grade level at which a gifted student is reading, for example, quantitative methods are best for measurements that can be represented numerically and somewhat discretely. If a gifted child in a regular second grade is tested as reading at a sixth-grade level, this finding helps teachers select reading material for that child that is intellectually more suitable, yet still age-appropriate emotionally, socially, and experientially. Quantitative measures are used in standardized tests of IQ knowledge, and academic achievement. Qualitative methods are more helpful for determining which content areas are an individual gifted student's favorites. Such methods also are useful for discovering a gifted student's particular learning style as well as the student's academic strengths and learning needs. Qualitative data collection methods such as interviews, observations, and self-reports can yield through profiles of each gifted student as a whole person, incorporating intellectual, psychological, emotional, social, and behavioral characteristics.

Describe some recommended intrinsic strategies for educators to help minority gifted students develop internal motivation, self-efficacy, and academic motivation. Summarize some recommended remedial strategies to help these students improve their performance in a specific area.

For intrinsic motivation and self-efficacy, educators should give minority gifted students consistent and constructive feedback, concentrate on individual students' interests, and provide them with choices. In addition, they should alternate their teaching styles to accommodate different student learning styles and furnish projects, simulations, internships, case studies, role plays, and other activities that promote experiential, active, hands-on learning. Educators can provide mentors and role models for students, as well as relevant, inspirational biographies. They can also use bibliotherapy. Teachers should work to establish classrooms with affirming and nurturing atmospheres for minority gifted students and all other students, as well. Teachers should use multicultural instruction that is meaningful personally, relevant culturally, and promotes self-understanding and insights. Some remedial strategies to improve student performance in specific deficit areas include counseling, tutoring, teaching study skills, test-taking skills, organizational and time-management skills, assigning journaling, making learning contracts, and teaching individually and to small groups.

Identify some characteristics of good instructional programs in writing for students gifted in the language arts.

For students who display gifts in the language arts, good writing programs should focus on developing the students' skills in expository writing, i.e. informational, descriptive, or explanatory writing; and in persuasive writing, i.e. argumentation, logic, and rhetoric. Teachers need to help gifted students learn the process of writing by concentrating on the components of developing ideas, opinions, and arguments regarding current issues; writing drafts, making revisions, and editing. Educators also should give their gifted students exposure and practice in writing narrative, poetry, and other forms of literature by presenting them with literary models of each form and then assigning them to write their own original work in each form. Students at the high school level can be assigned exercises wherein they copy the styles of their favorite authors. Such an activity helps them to develop control over the medium, develop flexibility and versatility, and eventually develop and identify their own individual writing styles.

Respond to the concern expressed by some parents and educators that acceleration of gifted students can hurt them academically by referring to research findings.

For those who worry that accelerating a gifted student will cause academic harm, they should be reassured that the majority of research shows that accelerated students get higher grades than gifted children who are not accelerated. Also, their grades compare positively with grades of the older students in the class. Researchers find that gifted students who accelerate report greater enthusiasm for and interest in school as well. Another academic concern is that students who skip grades in a subject or skip whole grades will have gaps in their knowledge. This does occur, but less often than people think because regular curricula contain so much redundancy. Gaps also do not present substantial problems for gifted children as they learn more quickly and thoroughly and can catch up more easily. Accelerating gifted students should not be penalized for unfamiliarity with any missed material, but should be allowed to cover it.

Describe some procedures for a gifted IEP(GIEP) team to follow if the GIEP does not seem to be working and/or if the gifted student has problems in school.

GIEP meetings exist in order to review the efficacy of the plan, not the student. Accordingly, "The Plan fails the child; the child does not fail the plan." (McIntyre, 2004). Some students may have had a GIEP for years that has not met their needs. If a student is in danger of failing in any subject or in school, the GIEP meeting should be reconvened. GIEP team members discuss the problem, appraise the circumstances, discuss available alternatives, and arrive at a solution for the individual student, changing the GIEP accordingly. The team can even write a provision into the GIEP for problems, specifying that if the student is potentially failing a subject, is unlikely to meet a GIEP goal(s), or his/her performance becomes substantially worse, the GIEP team will reconvene, including the teacher of the subject involved, and the GIEP will be modified to address the problem(s).

Regarding a student's Present Levels of Educational Performance (PLEP) for developing a gifted IEP, indicate the ideal general type and schedule of assessment and what more often happens in reality at public schools. Give an example of a measure often used wrongly in public schools to indicate PLEP and/or develop the gifted IEP.

Generally, objective assessments are needed to establish a student's PLEP. While subjective teacher observations, or cheklists showing students strengths and weaknesses in the curriculum, are helpful for developing a gifted IEP, they should not be used to define PLEP, which requires objective measurements are taken at least annually, or more often as needed. However, in the reality of public schools, this schedule is seldom followed with gifted students. Gifted IEP teams often wrongly use report cards, which measure learning of material taught in the past than in the present, and do not indicate student ability - a student may get As in a class which is still three grade levels below his/her capacity, or a student may be so uninterested in this lower level as to get Cs or even Fs. Either way, the student would not be functioning at his/her present level of educational performance.

Give examples of how teachers can enhance gifted instruction via educational games, and evaluate that instruction via assessment rubrics, both found on the internet.

Gifted and talented students tend to enjoy activities that they find stimulating and challenging. One way in which teachers can enhance their education is by offering students online educational games they can play wherein they apply the concepts they have learned in a lesson or unit. Online gaming has become so popular that it is a good choice for gifted students to extend and apply their knowledge and skills while having fun. For example, the About.com website has action games that address the content of various school subject areas. The Education World website's Online Game Archives has such games for students as Wacky Wordplay and Math Bingo. For teachers to evaluate student progress toward learning objectives they have developed for a unit, and evaluate the objectives themselves, they can find free assessment rubrics at websites like RubiStar. Teachers can customize RubiStar's rubric templates, including for assessment of virtual learning lessons/units.

Explain why homework is a recurrent issue with gifted students. Give examples of solutions.

Gifted children commonly start out in regular classes. Even after identification as gifted, the majority of gifted students are in regular classes most of the time with a small portion in supplementary gifted services like pull-out programs. They can end up with twice the homework: regular class assignments, plug gifted education assignment. Even students receiving accommodations in regular classrooms can receive double homework unless specific accommodations substitute gifted for regular homework. Having doubled work feels punishing, making students resent being identified as gifted. The The excess can mean not doing all homework, damaging grades. Solutions include assigning more advanced work taking the same time but applying lesson knowledge instead of repeating it, which gifted students often do not need. For example, a student could do ten advanced math problems instead of 100 rote problems. Instead of writing simple sentences including spelling words, a student could write a poem or short story using them.

Explain why gifted students' education must be addressed in both gifted and general education programs. Define the school district's responsibility in this matter and parental options in cases of district noncompliance.

Gifted children do not have gifts only part of the time; they are the same individuals all the time. As a result, their educational needs also are constant. Such rationale also extends for addressing those needs throughout the school day/year. Each school district has the responsibility not only to place a gifted student in a gifted program, but also to make whatever accommodations and modifications in the regular education setting, curriculum, and instructional methods are necessary and specified in the gifted IEP (GIEP) to meet the student's individual educational needs. Parents should know that if district administrators tell them a gifted pull-out program is the "only" accommodation available, this constitutes a severe misrepresentation of the district's responsibilities and the student's and parents' rights. Parents should inform administrators they are aware of these responsibilities and rights and should hold administrators responsible for addressing and fulfilling these rights. Parents can advocate for their child at GIEP meetings and/or request due process hearings.

Discuss some common emotional characteristics of intellectually gifted children.

Gifted children often display emotional intensity by experiencing and expressing extremes of emotions. They likely exhibit more emotional sensitivity than other children. They may feel more anxiety, guilt, and sense of responsibility than other children their age. Despite their superior abilities, they can feel inadequate or inferior due to their extreme sensitivity, high standards, perfectionism, and tendency to internalize. They may be shy and timid, and as a result they may be lonely. However, they are also likely to show concern for other people. Their sense of right and wrong is often heightened, prompting strong reactions against dishonesty, hypocrisy, and injustices. They tend to retain stronger memories of emotional experiences. They may have difficulty adjusting to changes. Because of their emotional sensitivity, they are susceptible to depression. They tend to have a need for security. They may suffer from physiological symptoms of emotional states, such as getting a stomach ache from feeling anxious.

Describe some examples of enrichment activities for young gifted children that develop skills in forming and testing hypotheses.

Gifted children typically exhibit advanced abilities in conceptual thinking. Their skills should be developed with activities wherein they manipulate information rather than merely memorizing or recognizing it. To stimulate and practice hypothesizing, you can ask questions of a gifted child beginning with "What would happen if we..." For example, what if we put a block into a glass filled with water? What if we put a glass over a lit candle? If we put tennis balls into a shoebox, how many could it hold?, etc. Then you can test these hypotheses by following through and performing the hypothetical action to see what happens. These activities suit children in Piaget's Concrete Operations stage. For children who can think abstractly without concrete objects, activities can be extended to imaginary events and fantasies, e.g. "What would happen if you could fly?...if you were invisible?...if snow were ice cream?...if you were 12 feet tall?", etc.

Discuss some considerations and recommendations for educators to support the affective development of gifted students.

Gifted education programs, both inside and outside of general education classrooms, should incorporate curriculum elements designed to support emotional, social, and career development in gifted students. At all age/grade levels, school curricula can be proactive by including psychoeducational knowledge regarding the way in which giftedness influences these developmental domains. When teaching social sciences and literature, teachers can give assignments related to the psychosocial aspects of these subjects. Classes can have group discussions about developmental challenges. Career and talent development often present issues for gifted children at much younger ages than for other students, so these topics should be addressed in the curriculum. Viewing and addressing underachievement and high achievement from a developmental perspective can help both students and educators. Teachers should affirm student resiliency and personal strengths, which can be eclipsed by performance /underperformance. Advocates for legislation, funding, and services for gifted students also should emphasize affective considerations in their efforts.

Describe some of the characteristics of the profile of a "challenging" gifted student.

Gifted students described as "challenging" are "divergently gifted". In other words, they are highly creative and unconventional. Schools often do not identify such students for gifted programs unless such programs have been implemented for a sufficient number of years and teachers have received enough in-service training. These children do not learn to use the system to their own benefit as more "successful" gifted students do. They tend to question authority and challenge teachers in class. Consequently, they are rarely rewarded for their abilities and often experience conflict. Those children who challenge their peers are not accepted, while others are liked for their originality and humor. They can be disruptive in class due to their spontaneous behaviors. Frustration and low self-esteem are risks for these students. Additionally, they may risk delinquency, drug use, and dropping out of high school if they do not receive suitable intervention by the time they reach middle school/junior high school.

Describe some enrichment skills in sequencing and chronology important for gifted students to develop and use in social studies classes.

Gifted students in social studies classes should become familiar with and apply the terminology used for chronology and time elements. Given a series of historical or recent events, they should be able to arrange them in the correct chronological order. They should become familiar with timelines, learn how to read timelines, and develop the ability to create their own timelines. They should learn how to research chronology and time. Gifted students should have or develop an understanding of the concepts of time, change, and continuity. They should have or develop enough facility with sequencing and ordering to enable using these skills to plan and carry out tasks assigned in the social studies class. These organizational skills related to planning and sequencing also benefit them in their other classes in areas such as accomplishing academic tasks and in personal life in areas such as planning and time management.

Identify some examples of curriculum design features that teachers of gifted students should consider when selecting curriculum materials.

Gifted students need greater breadth and depth of knowledge and processes than are afforded by school textbooks. When selecting curriculum materials for gifted students, teachers should consider the following curriculum design features: 1 - The purpose and rationale of the curriculum design should be clear. 2 - Selected materials should address objectives for each lesson. 3 - Materials should include appropriate, stimulating, and challenging activities for gifted students. 4 - Materials should include instructional strategies suitable for gifted students that teachers can use. 5 - The materials should include valid and reliable assessment procedures teachers can use to assess the effectiveness of instruction using those materials. 6 - References for additional materials and resources should be provided. 7 - Ideas for extension of the curriculum. activities, and materials should be given. 8 - Technology should be incorporated in the materials. Today many materials are available on websites, which are most likely to adhere to the preceding guidelines.

Describe how social studies teachers can encourage gifted students to develop critical thinking skills through their reading of subject content.

Gifted students need to develop their skills for analyzing, synthesizing, and evaluating subject content that they read. Students need to realize that anything in print was written by someone. Teachers can help their gifted students discover who wrote or published the essay, article, textbook, eyewitness report, or primary document they are reading. Furthermore, teachers can ask the students to ascertain the author's originally targeted audience, why the author wrote the material, and what purposes the author hoped to accomplish by writing it. Teachers can ask their students to draw inferences about the author's intended audience. Such an activity will help the students learn how to select the best research sources for their own class and independent study projects. Developing their skills in critical reading will help students develop their general critical thinking skills, as well.

Describe the rationale and process of pre-testing as one type of accommodation that can be made in the general education setting to provide specially designed instruction for gifted students.

Gifted students often have mastered the content of a lesson or subject at their grade level before teaching has begun in that class. Making them sit through instruction in material they already know only bores and/or frustrates them and damages their motivation; moreover, they are prevented from learning new things in the subject at a higher level and at their own pace. The first section of the gifted IEP (GIEP) specifies the student's recent Levels of Educational Performance. When the GIEP team determines these, they can determine the need for pre-testing. Student whose pre-tests demonstrate mastery of material can be given alternate assignments that challenge them and allow them to learn new material. Students whose pre-tests demonstrate they have almost mastered a subject can be given only the parts they have not mastered to study, thereby saving time and redundancy. These students can use the rest of their class time for independent work on alternative assignments.

Describe some skills gifted students need related to the use of maps or globes, graphs, and images in enriched social studies education.

Gifted students participating in enriched social studies courses need to learn how to read maps and read, understand, and apply map legends, scales of miles, and mapping symbols. They need to be able to read and use compass roses, grids, and other mapping tools. They should be able to understand and apply time zones. They need to understand the concept of distance. In addition, they should be able to draw inferences from maps; to compare various maps, to analyze and interpret different kinds of maps, and learn how to create their own maps. Students in enriched social studies courses should be able to read and interpret charts, tables and other graphics. They also should demonstrate the ability to interpret the meanings of cartoons, photographs, paintings, drawings, and other images relative to social studies topics. The ability to interpret visual images such as pictures and other graphic representations is also an important skill of analysis.

Identify a number of research and writing skills that gifted students should have in enriched social studies courses.

Gifted students should acquire and demonstrate a number of skills for doing research, writing up research results, and writing in general. Students should learn how to acquire the information they need. In addition, the should learn different note-taking practices, how to organize information, and be able to identify primary and secondary sources of research and know how to use them. They should be able to read textbooks with comprehension, and they should have/cultivate the ability to look for patterns in the material they read. They should also be able to interpret the information they encounter in social studies, and then analyze, synthesize, and apply the information to different circumstances and real-world situations. They should be able to support their viewpoints using pertinent documents and facts. They should learn to create bibliographies and webographies. Finally, they should recognize what is important in research and writing.

Give some examples of how a social studies teacher with gifted students can help them learn to identify and analyze writing styles in assigned reading material and apply them.

Gifted students should understand and apply both the content and the writing style in their social studies reading. Teachers can show students various writing approaches in written materials. For example, some authors first will establish the context for a historical event and then recount its details in chronological order of occurrence. Others may open by relating an anecdote and then explain how it illustrates or is related to the topic. Reporters often use a common journalism technique of opening an article with a paragraph summarizing the key points of the report and then filling out these key points in subsequent paragraphs with more details and/or quotations of comments or responses from involved people whom they interviewed or those persons they consulted as expert sources. Teachers can stimulate discussions by asking student to project how each of the styles might influence different reading audiences. They can also provide challenges to gifted students by having them apply the styles in their own writing.

Generally explain how and why school counselors and counselors in general need preparation specifically for working with gifted students.

Giftedness has significant impacts on the emotional and social development of young people. They encounter the same universal developmental transitions and challenges as all children, yet they are likely to experience them quite differently. In addition, they experience other, individualized developmental issues. They will be concerned with moral, academic, and career issues at younger ages than other students. They may also have concerns that other students do not experience at any age. The issues that concern these gifted students have ramifications for their overall wellbeing. Thus school counselors and outside, private counselors must possess knowledge regarding the characteristics of gifted students. Knowing about the emotional intensity and hypersensitivity of many gifted students keeps counselors from misinterpreting these as abnormal. Knowledge of gifted students' characteristics can explain classroom and/or social problems and afford more perspective on developmental issues. This knowledge also should inform counselors' case studies and intervention plans.

Differentiate between goals and objectives in a gifted student's IEP, and explain how they are related.

Goals are the long-term targets for a student's education, while objectives represent short-term targets, but this is not the only difference. Goals need not be specific or measurable. While goals are more general and global, they still must be specific to the individual student rather than assigned to that student based on his or her class or group. First, identifying the individual student's educational needs and the final outcomes desired are the ways of determining the student's educational goals. Short-term learning objectives then specify the smaller steps the student must achieve toward eventually meeting the associated goal. Objectives are also the IEP team's means of measuring student progress toward a goal. Objectives quantify criteria for attainment and state when these criteria are to be met. Goals and objectives should be congruent with one another: when read together, they should "make sense" to parents as well as to educators.

Describe some general goals of a good gifted education program.

Good gifted education programs seek to facilitate the gifted student's mastery of basic reading and math skills at paces and depths suitable to the student's abilities; to foster abilities of reasoning and critical thinking; to create an environment conducive to divergent thought; to promote inquiry; to promote challenging attitudes relative to learning; to develop higher level skills in spoken and written language; to develop skills and methods for doing research; to cultivate students' comprehension of issues, problems, themes, and systems of knowledge that structure the extrinsic environment; to foster student development of self-understanding; to enable learning experiences outside of school that fit the individual student's educational needs; to provide or improve opportunities for future student development and planning; to develop creative thinking and problem-solving skills; to foster effective social skills for interpersonal interaction and coping; and to develop the student's skills of metacognition for self-direction and independent learning.

Generally describe the relationship of the general education teacher to the Gifted IEP (GIEP) written for a gifted student who attends their class, including considerations for the student's parents.

If a gifted student is in a general education classroom for any part of the school day, the general education teacher needs to familiar with the content of the student's GIEP. None of the privacy laws protecting the privacy/confidentiality of student records applies to either general or gifted education teachers' access to the GIEP. Some school districts have their own privacy procedures, such as having general education teachers sign a log to read a GIEP, but teachers are still allowed to read the GIEP. They cannot align with the GIEP's goals, objectives, activities, and materials without knowing them. Familiarity with the GIEP facilitates general teachers' coordinating their instruction with gifted teachers/programs. If parents believe their child's teacher does not know about the GIEP, and/or that their child is identified as gifted, they should ask the teacher. Bringing their child's gifted written report and GIEP to parent-teacher conferences is advisable.

Describe some general stages in problem-solving a group of gifted students might use as a team. Give one example of an established model for creative problem-solving and its components.

If a group of gifted students wants to solve a problem as a team, the first stage they should address involves understanding the nature of the specific problem. That accomplished, in the second stage they would brainstorm ideas for potential solutions to the problem. Once they had a number of possible solutions, the third stage would involve deciding what to do. Dr. E. Paul Torrance, who developed the most widely used standardized tests of creative thinking, founded the Future Problem Solving Program (FPSP) in 1974. Today it is used with hundreds of thousands of students, nationwide and in several other countries. The FPSP model contains three sections and six steps: Section 1 - Understanding the Problem Step 1 - identifying challenges in the problem Step 2 - selecting an underlying problem Section 2 - Generating Ideas Step 3 - producing solution ideas Section 3 - Planning for Action Step 4 - generating and selecting criteria Step 5 - applying criteria Step 6 - developing an action plan

Identify some criteria to consider for accelerating a gifted student in school, as well as steps to take in that process.

If a student's scores on standardized achievement tests are many grades above age/grade level, or are so high that they are off the tests' charts, the student is a candidate for acceleration. If a formerly enthusiastic student begins exhibiting boredom and/or behavioral problems not due to other factors, this is another criterion. The student, parents, and school personnel ideally should all agree that accelerating would benefit the student. Early in the process, the IEP team and school psychologist should be involved. Classroom teachers should be consulted, as well as others who know the student best, e.g. the school's gifted and talented coordinator, principal, and/or guidance counselor. The school psychologist should evaluate the student relative to acceleration. Parents can consult state Department of Education coordinators of gifted education. Support from involved teachers and coordination and continuity from school officials is important. Acceleration decisions are usually made by the principal and/or IEP team/committee.

Describe examples of some provisions that a school district's written whole-grade acceleration plan should include for a gifted student to skip grades in school.

If the school has determined a gifted first-grader can do third-grade work, the written whole-grade acceleration plan should include such provisions as these: The third-grade teacher should report and discuss any signs of academic and/or emotional stress in the student to the school staff person assigned to monitor plan implementation. The monitor should check with the third-grade teacher weekly on student progress. A gifted intervention specialist and/or third-grade teacher should obtain/develop curricula and assessments that compact only the second-grade standard the student has never experienced to make up anything he will miss by skipping second grade. He will complete these standards/skills during free time at school and/or home with his parents helping him as needed to master them. Upon successful transition, he remains in the accelerated grade until additional acceleration options are indicated. By middle school, the student's program should be reviewed to see if dual-enrollment options might benefit him.

Explain how teachers can help gifted students relate arts and sciences in an integrated curriculum by giving an example involving perception, physical reality, and illusion.

In both the arts and the sciences, we encounter illusions wherein things are not really as they seem. Optical and auditory illusions can be created and explained in the sciences, and artists often deliberately create illusions to communicate a specific idea with their respective audiences. Teachers can expose their gifted students to various styles of visual art such as Impressionist, Surrealist, and trompe l'oeil techniques; to experimental art; to performance art such as modern dance performances; and other presentations. They can have students write down their observational analyses of the artists' assumptions about the nature of physical matter, e.g. whether matter is static or in perpetual motion; whether it is solid or not; its weight, speed, etc. Students then can use the laws and principles of physics to examine these assumptions. Finally, teachers can assign students to write essays challenging or defending the artistic renderings.

Explain the functions of some components of curriculum writing and how these components can help gifted students learn. Give one example of a rubric that can be used with any teaching technique.

In curriculum writing, the stated instructional objectives and the specified student activities are concerned with the processes student use in constructing knowledge ( for example, analysis, synthesis, and application). These processes are both cognitive and affective in nature. When curriculum writers include both variety and balance among these processes in the activities they design, they enable gifted students to practice using a fuller range of encoding, decoding, and recoding processes involved in learning new ideas, principles, and information. Instructional strategies represent another area wherein curriculum writers should provide variety. Teachers can use class discussions, students dramatizations of topics, and independent study projects, for example, affording gifted students different ways to learn and utilizing broader ranges of abilities and learning styles. The Multiple Menu Model of curriculum development's Instructional Sequences Menu is a rubric teachers can use with any teaching technique/strategy to help them present lessons in the most effective order.

Regarding the selection of representative topics for a curriculum, compare and contrast some features of traditional methods with those of more progressive models of curriculum development.

In developing curricula, teachers must select topics that illustrate a subject's fundamental principles and characteristic concepts. Both traditional models of curriculum development and more progressive ones, such as the National Research Center on Gifted and Talented's Multiple Menu Model of curriculum development (Renzulli et al 2000) take into account the students' ages, maturity levels, prior knowledge, and levels of experience. These methods differ in that traditional instructional models typically require teachers to cover a whole textbook with their classes by the end of the the term or year; however, the Multiple Menu Model asks teachers instead to narrow down, from all potential texts, chapters, or other sources of information, those few that best represent the concepts and principles of the subject area. This model's authors recommend using a three-phrase approach to choosing content: intensive group coverage, extensive group coverage, and intensive individual or small group coverage.

Describe the category of experimental research in terms of how gifted students can use it in independent study projects, including examples.

In experimental research, the researcher seeks to discover whether one variable causes changes in another variable. Statistical methods of multivariate analysis can also be applied to study the effects of and on multiple variables within the same experiment or set of experiments. For example, gifted students use independent study projects to research whether a plant responds differently to various small stimuli or which combinations of substances will produce a certain chemical reaction. Experimental studies involving human subjects include such research questions as whether the amount of sleep the night before an exam affects student test performance, whether one school curriculum is more conducive to student motivation than another, or whether internal or external rewards have greater positive influences on student performance. Experimenters establish a treatment group receiving the intervention being tested, and a control group receiving no treatment to control for extraneous variables. By manipulating the independent variable being tested, researchers can see if it affects the selected dependent variable(s).

Characterize generally what the gifted IEP is and is not for the teacher. Contrast how goals vs. short-term learning objectives are developed in the school year and how teachers and students can collaborate in developing objectives.

In general, the gifted IEP is a guide for the teacher to help focus instruction on the student's educational needs and goals. It is not a rigid directive forcing the teacher to adhere to specific activities determined months before instruction. At the beginning of the school year, the student's gifted IEP should establish the student's main educational goals and a schedule within the school year for their attainment. Because goals are more general than short-term learning objectives, they allow time for teacher and student to collaborate in determining a school subject for a given goal that is compatible with the student's abilities and preferences. After arriving at a subject, the teacher and student can collaborate further in developing individual short-term learning objectives according to the schedule previously established. The objectives specify the smaller increments/steps toward achieving a goal, permitting the teacher to adjust to the student while assuring progress.

Provide examples of things parents should avoid with their gifted children in order not to interfere with developing the child's gifts.

In general, toddlers are prone to tantrums due to frustration. Gifted toddlers are even more likely to demonstrate this characteristic when their cognitive development outpaces their physical development and they can conceive of actions but cannot perform them. Parents should not punish such tantrums as they allow emotional venting and reduce tension. Researchers have found that parents who are overly directive and controlling tend to influence children to become less exploratory and curious, less independent and self-directed, less internally motivated, and less creative. Total permissiveness is not ideal, since children need appropriate limits, boundaries, structure, and guidance. Detached or uninvolved parents are not helpful. Parents should avoid punishing or disapproving of gifted children's actions as abnormal just for being unusual or unfamiliar. Parents should not avoid activities with their gifted children from insecurity regarding their own preparation or competence. Activities confer important bonding experiences as well as children's learning and positive reinforcement.

Describe the Torrance Tests of Creative Thinking, including Torrance's main focus, abilities measured, sample test items, and the validity of research findings.

In order to operationalize creativity for research purposes, E.P. Torrance focused on its problem-solving aspect. His tests include nonverbal and verbal forms. "Thinking Creatively with Pictures" features three activity groups: namely, drawing lines elaborating on one shape, drawing line finishing an incomplete picture, and drawing as many different pictures as possible using the same shape. "Thinking Creatively with Words" features six activities involving generating questions, alternate uses for things, and guesses. All these activities are timed and scored for fluency, flexibility, and originality. In addition, the nonverbal forms are scored for elaboration. Torrance's tests apply to grade levels from Kindergarten through graduate school and are the most widely used creativity tests. They also have the most research supporting their validity worldwide, as they have been translated into many languages. While their longitudinal predictive validity is not high (.62 for males, .57 for females), Torrance indicates they match or exceed rates for intelligence measures.

Describe some considerations for developing and IEP for a gifted student relative to the general education classroom setting.

In part because federal law addresses special education needs for disabled students but not for gifted students, in part because emphases on inclusive education, and for other reasons, most public schools do not have many (or any) classrooms with all gifted students. Moan schools have pull-out gifted programs, which comprise a very small part (about 5%) of the gifted students' school hours. Thus the general classroom should be the focal area for modifications or accommodations included in a gifted IEP. In addition, if any school problems arise in gifted students, such problems are likely to arise in regular classrooms. Gifted students who learn more quickly and/or with less repetition can become bored and/or impatient with the class pace and come to dislike school. Although such problems may be minor in elementary school, they should still be addressed early to prevent the development of larger problems by high school. Simple general classroom changes in gifted IEPs can make tremendous differences.

Describe the rationale for independent study agreements as one form of accommodation in specially designed instruction, and how these agreements work for gifted students.

In public schools, gifted students are not commonly enrolled in completely separate gifted programs/"tracks" where all classes differ from general education classes in order to accommodate the gifted students' learning needs. Such gifted students most often are placed in general education classrooms for part/all of the school day. They may have pull-out programs for gifted instruction, but such programs represent a minority of the student's school day/week. Due to asynchronous development, some gifted students are farther ahead of of the regular curriculum in some subjects than in others. also, some gifted students still may find the regular curriculum relevant, but they simply may finish their classwork sooner. Independent study agreements offer gifted students ways to occupy their free time instead of waiting for the rest of the class to catch up. They can work on one big project incrementally, or on a series of smaller projects. Such projects should be designed to meet goals written in the student's gifted IEP.

Discuss the merits for students gifted in subjects like math of working with other similarly gifted students relative to the idea of educational inclusion.

In recent years, the federal government has passed legislation guaranteeing students with disabilities a free, appropriate public education, including special education services as needed to accommodate disabilities in order to provide this education. This legislation has led to great emphasis on inclusion in education. However, the laws do not address students with gifts. Educators still apply the notion of inclusion to gifted students in that they should experience working with non-gifted students. This is true, but it is not a big concern as most gifted students in public schools are in regular classrooms most or all of the time. Equally important is the need for gifted students to be able to work with similarly gifted students. Such collaborative work not only promotes their cognitive development. It also contributes to their emotional and social development through sharing ideas, debates, and constructive disagreements and through decreasing the social isolation that many math-gifted children often experience.

Define the statistical terms reliability, validity, internal validity and external validity, giving examples related to gifted education.

In scientific research, reliability refers to whether a measurement can be repeated with consistent results, regardless of the administrator. For example, a reliable standardized achievement test given to the same gifted student should yield similar scores each time, even when different teachers/personnel administer the assessment. Validity refers to whether an instrument or procedure measures what it claims to measure. For example, a standardized achievement test is not a valid instrument for measuring IQ; conversely, a standardized IQ test is not a valid instrument for measuring academic performance. Internal validity refers to whether the finding of an experimental study are attributable to the intervention or treatment used in the study and not to any other variables. For example, students could test better due to being taught a different curriculum or to having a different teacher. External validity refers to how much a study's finding can be generalized to populations beyond the study's sample.

List a number of comprehension skills included in enriched social studies skills for gifted students.

In social studies, students should be able to attain and demonstrate the following: recognizing cause and effect relationships in social studies topics; making comparisons and contrasts among concepts presented in social studies; making connections between events, persons, things, etc.' evaluating the subject content in social studies; paraphrasing the social studies content they have learned and showing thorough understanding; differentiating between facts and opinions in social studies; drawing inferences from social studies material they learn; drawing conclusions about a topic in social studies; locating and solving social studies problems with multiple steps and; managing and understanding various interpretations of social studies events or issues; and making decisions.

Describe some examples of rules that a school district's testing policy might include for gifted students accelerating in one subject.

In some districts, rules differ for a student's first and subsequent years of subject acceleration. In the first year, if a test exists at the accelerated subject grade level, testing is required for students whose overall/age-level grade is third-grade or higher; and the accelerated subject is reading or math. Testing is more likely optional if the accelerated subject is science, social studies, or writing; and if the student's overall level is second-grade or lower and the accelerated subject is third-grade or higher. If no test exists at the accelerated grade level and one exists at the student's overall grade level, testing may be optional. In subsequent years of acceleration, if a test exists at the accelerated grade level, testing is usually required. If no test exists at the accelerated-subject grade level and one exists at the student's overall grade level, districts may prohibit testing.

Define the terms standard deviation and variance and explain how they are related to statistically normal distributions and to standardized IQ test scores in gifted education.

In statistics, standard deviation shows how much the numbers in a given set vary within their distribution and how close to or spread away they are from the mean/average. Standard deviation is the square root of variance. Variance is calculated by squaring each variation from the mean and averaging the squares. Standard deviation indicates how much variation around the average is expected; greater variations are statistically significant. In IQ scores, the most common standard deviation is 15 points; variations of +2 standard deviations, i.e. 30 points above are below the mean, are statistically significant. Thus, a student scoring 134 does not necessarily have a higher IQ than one scoring 130 on the same IQ test. However, a students scoring 160 compared to another scoring 130 on the same test shows a significant different of 30 points, or two standard deviations. This difference is sometimes used for labeling levels or degrees of giftedness.

Identify some important approaches to differentiated instruction for gifted students in the language arts. Comment on teacher expectations relative to these differentiations. Give an example of curricular adaptation to meet gifted student needs.

Included among differentiation approaches is adapting language arts curricula for gifted students are acceleration; depth, complexity, challenge, and creativity. In differentiating the curriculum for gifted students, teachers must adjust their expectations regarding the demands of content, process, and concept to higher levels. To meet these higher expectation levels, one method is to give students access to more advanced content or curricula when they are younger while still assuring that they can meet prescribed standards at all levels below that. Students also may be allowed to accelerate through the standard curriculum. Either way, teachers must adjust their expectations for advanced student levels. To meet gifted students' needs for advancement, depth, and complexity, one example is: following a class discussion on major themes in novels, a teacher could assign a gifted student to select a novel and write an essay examining how its major themes are treated in one chapter.

Identify some benefits that independent study confers to gifted students, to their teachers, and to underachieving gifted students.

Independent study gives gifted students encouragement and preparation for initiating, implementing, and successfully completing their own scholastic activities in their areas of interest. As such, it provides them with a structural context for making choices and decisions about what and how they learn. It also teaches them how to communicate what they have learned. When gifted students have independent study projects, these also benefit teachers by helping them to fulfill the roles of facilitating students' progress, listening to students, and clarifying ideas and objectives. Many gifted students underachieve; this can be related to lack of interest in assigned topics, frustration at not being able to study objects in more depth, boredom with the mainstream class pace and/or types of assignments. Independent study of a topic a student is passionate about can alleviate these factors and increase task commitment. Planning and executing projects stimulates inquiry, builds motivation and self-discipline, improves study skills and habits, and promotes positive attitudes and self-concepts.

Describe some purposes and positive outcomes of independent study for gifted students.

Independent study projects allow gifted students to explore a variety of subjects and areas in which they are interested. Planning, developing, and carrying out an independent study project can serve as the student's introduction to the methods used to organize, research, and present the results obtained to others. With guidance and support from teachers, independent study can encourage gifted students to develop individual qualities of initiative as well as their intellectual and creative abilities. Independent study activities stimulate gifted students' skills in critical thinking, logical reasoning, and continuity in focus. By working on independent study projects, gifted students develop a sense of personal responsibility for pursuing their goals. In addition, when they pursue and successfully attain their goals through independent study, gifted students attain a sense of satisfaction and achievement.

Describe some typical strengths of intellectually gifted learners, and provide a potential "problem" behavior corresponding with each of these strengths.

Intellectually gifted students often have advanced organizational skills. This advancement benefits them academically, since it facilitates learning. However, this advancement also can manifest in these students exhibiting a need to organize things and people beyond their own academic subjects. Positively, this characteristic contributes to leadership qualities; negatively, it contributes to making rules to complex for others and to being perceived as being bossy. Intellectually gifted student often possess knowledge ahead of their peers in both level and breadth; they frequently have larger vocabularies and more facile use of their respective vocabularies. Problems associated with these strengths include boredom with school and classmates and the potential for using language to manipulate others. Intellectually gifted students tend to have high expectations both for themselves and others. While this strength is a motivational quality, its drawbacks include perfectionism, a lack of tolerance for their own and/or others' shortcomings, and a potential for developing depression due to disappointment over unmet expectations.

Describe some typical strengths of intellectually gifted learner, and give a potential "problem" behavior corresponding with each of these strengths.

Intellectually gifted students often learn and remember new information more quickly than other students. A potential side effect of this facility involves impatience with other less gifted, as well as with routine procedures they find superfluous and limiting. The curious/inquisitive nature of intellectually gifted students as well as their need to seek meaning can result in questions that embarrass others; others' perceptions that the gifted students are nosy and/or rude; and perceptions that the gifted students go overboard in pursuing their interests. Another strength involves internalized motivation; however, as a downside, these students can seem overly willful and resistant to direction from others. Students with superior capacities for synthesis and conceptualization tend to enjoy solving problems and dealing with abstract ideas. However, these students are more likely to question/challenge teaching practices and routines. They also tend to focus on truth and fairness, so they may have excessive humanitarian worries and/or difficulty accepting inequitable realities in life.

Give an example of a simple experimental research design you could use with a class or group of gifted students in a public school system in order to investigate teaching methods, identifying the terms random sampling, independent variable, dependent variable, manipulate, and control group.

Introduce two teaching methods, neither one normally used with the students. Divide students into two experimental groups and one control group by random sampling; i.e. they all have an equal chance of being in any group, preventing biased selection. One experimental group gets a lecture on a new topic followed by a question-and-answer/discussion session; the other gets a hands-on activity on the same topic followed by a question-and-answer/discussion session. The control group contains comparable students receiving no treatment; e.g., they receive the same teaching method normally used in your school, on the same topic as the experimental groups. The control group helps rule out the influences of extraneous/confounding variables on differing results between experimental groups, supporting the experimental treatments' differing influences. All groups receive the same pre-test and post-test on the topic. The teaching method is the independent variable, which was manipulated. The dependent variable is post-test score.

Comment on the relative status of educational services to gifted students from minority groups, including some supporting findings.

It has long been a concern of educators that gifted children with lower socioeconomic levels and those children with some minority cultures and/or languages have not been represented sufficiently in gifted educational programs. Such students also often have gifts not as easily evaluated by traditional measurement instruments. Thus, these students have not been afforded opportunities to develop and apply their gifts fully. In 1988, the National Educational Longitudinal Study found a significant level of under-service by gifted and talented programs to students from economically disadvantaged backgrounds. This study also found varying levels of service to different student minority groups. The National Academy of Sciences reported in 2003 that minority students were overrepresented in special education and underrepresented in gifted education. The National Research Council reported that 9.9% of Asian students and 7.47% of white students in America were placed in gifted programs, while only 4.86% of Native American, 3.57% of Hispanic, and 3.04% of African-American students were identified as gifted.

Summarize some of the finding regarding the personality characteristics of persons with various creative gifts, and state the implication of these characteristics for predicting creative behaviors.

Jane Piirto (Understanding Those Who Create, 1998) found that when compared to others with creative gifts, artists are more impulsive and spontaneous. The author found that writers tend more toward nonconformity than do the other creatively gifted types. In addition, Piirto found that in comparison to other creative types, musicians tend more toward introversion. She discovered that architects are not as flexible as other creative persons. Furthermore, she concluded that creative engineers and inventors were likely to be more well-adjusted overall than were other creative types. These findings imply that specific personality traits may need to be considered in order to make predictions, since different personality traits are associated with different domains of creativity. Kerr and Gagliardi (2009) find this domain-specific approach more useful than "...seeking one creative personality type that fits all creative occupations.: Indeed, the latter seems doomed to failure in light of the many types of creativity.

Identify the components of language study. Describe elements of language study that gifted students should be offered in language arts instruction, as well as a preferable instructional approach.

Language study includes the main components of vocabulary, grammar, and syntax (i.e. sentence structure and word order). Therefore, instruction in the language arts for gifted students should emphasize the development, extension, and correct use of vocabulary. Instruction should include promoting an understanding of etymology and the origins of words, including word roots, prefixes, and suffixes, and their original language sources. Language arts education for gifted students should include making analogies to teach an understanding of relationships between/among concepts and words. Gifted students also should learn linguistics, or the formal study of language; the history of their language; and semantics, or the study of the meanings of words. In teaching language arts to gifted students, the preferred approach integrates all of these elements of language study into one unified program for a holistic appreciation rather than teaching the elements separately on a disconnected piecemeal basis.

Describe some social and psychological factors contributing to underachievement in gifted student from minority backgrounds.

Low self-esteem in general hampers student achievement. Researchers also find that poor self-concepts specific to academics and social interactions interfere. Some researchers have emphasized the need to consider the strength and/or positivity of a gifted minority student's sense of racial/ethnic identity. Students whose racial identities are not positive can be more susceptible to counterproductive peer pressure. They also may associate academic achievement with the dominant culture, hence with betraying their minority culture, i.e. "selling out" or trying to "act white". This view results in less effort, which results in less achievement. Psychologically, with a high internal locus of control, a student attributes his or her achievement to ability and effort and is more likely to achieve. Minority students who expect external factors such as discrimination and injustice to block their success demonstrate a high external locus of control and are less likely to achieve up to their potentials.

Describe some common intellectual characteristics of young gifted children and give some examples of enrichment activities addressing these.

Many gifted children have stronger reasoning abilities than do other students, and gifted children are better able to understand abstract concepts at earlier ages. Also, they often do not need to reinforce what they have learned with as much repetition as other children need. They should not simply be asked to recite or repeat something they have learned. Instead, they should be asked to use the higher cognitive skills they possess. For example, for comparison-contrast, you can ask the child the similarities and differences in words familiar people, characters in a story, familiar songs, holidays, seasons of the year, birds and butterflies, etc. For categorizing, ask the child to group things, e.g. books, foods, toys, clothes, friends, or feelings. For sequencing, ask the child what happened in the beginninhg, middle, and end of a story. For summarizing, ask the child the most important events during a party, trip, or other activity with varied experiences.

Discuss a side-effect of accelerating gifted students on their perception of their status relative to the class or group and how adults can address this perception. Discuss some considerations related to accelerating in a single subject.

Many gifted students in regular classes at their age level perceive that they are the smartest student in their class. When they are far enough beyond grade level to accelerate, the student often finds after acceleration that they are not the smartest in the group anymore. This change in status can require considerable adjustment, especially for students who have been accustomed to everything being too easy for them. In this situation, parents and teachers should not pressure students trying to adjust to unfamiliar circumstances. They should give them emotional support rather than pushing too much too soon for academic performance. They should also make clear to the student that a choice to accelerate can always be reversed if it seems unsuitable emotionally, socially, and/or academically for them. Adults should also help gifted students understand that reversing acceleration does not represent a failure on the part of the student.

Describe some typical strengths of intellectually gifted learners, and provide a potential "problem" behavior corresponding with each of the these strengths.

Many intellectually gifted students are academically independent. Superior abilities for learning, knowing, and understanding afford them both the skills and the confidence for autonomy and self-direction in learning. These skills and confidence, plus their thinking and learning differently from their age peers, promote self-reliance in such students. They are likely to prefer individualized tasks. Because of these factors, these students may be seen by others as not conforming to accepted norms. They also may reject input from their peers, teachers, or parents, and these other individuals likely will not receive such rejection well. Greater amounts of versatility and diversity in their abilities and interests give these students more options, but these characteristics also can cause time-management issues and cause a disorganized impression. Strong senses of humor can be misconstrued by age peers, and as a result these students may evolve into "class clowns". Their tendencies to seek causal relationships can make them uncomfortable with irrational/illogical feelings or traditions.

Discuss some aspects of the issues of challenge and frustration in the education of students gifted in mathematics.

Many mathematically gifted students enjoy challenges that engage their superior abilities for calculating and solving problems. However, gifted students who only have experience with effortless work become uncomfortable confronting challenges and/or frustration inherent in learning more difficult material. In addition to teaching advanced students, teachers are charged with the job of helping them realize that confining their education to easy pursuits deprives them of many rewards, and a life without struggle leads to less achievement and fulfillment. Teachers can give students examples illustrating challenges to adults. When educator would tell his status about the time he did not calculate whether his rope was long enough to repel off a cliff and reach the ground, and he had to jump 15 feet. He also gave them problems to work where mistakes were common so they could see that making errors was not the end of the world.

Identify some issues with gifted education and IEPs that many parents of gifted students commonly experience across public school districts.

Many parents of gifted children have commonly expressed several repeated experiences across public school districts. First, many parents report being told that their children must make up assignments in the general education classroom they missed due to the gifted pull-out program. Furthermore, some parents are told that since their child earns As in school, she/he needs no special attention or supplementary educational services. Parents often express concern that their child is growing comfortable with levels of schoolwork too low for his/her ability, that school is not offering challenges to their child, and/or that their child is developing bad study habits. Many parents are surprised and disappointed that their school district seems averse or avoidant to acknowledge and address their child's giftedness, since parents often expect for school districts to welcome superior learners. Parents often express perplexity or resentment at not knowing about available educational programming for their gifted children.

Discuss a disparity between the approaches of many math programs for gifted students and teaching methods that would be more truly motivating of passion for mathematics. Give some examples of more motivational activities.

Many school programs for mathematically gifted students are designed with the goal of moving them through the curriculum as rapidly as possible. However, this approach of fast advancement does not cultivate a student's interest or passion for math. Zaccaro (2008) finds this analogous to teaching aspiring musicians "all scales and no music." Students made to practice scales but never allowed to play music that moves them emotionally will not develop passion for it. Practicing the structural fundamentals is necessary for mastery, but these fundamentals should not be taught exclusively. The same is true of math; students need to be given opportunities to work with material they find exciting and wondrous. Students can experience how the earth's circumference was determined 2500 years ago using simple geometry, apply knowledge of the speed of light to realize that viewing stars is time-traveling to the distant past, calculate ship-to-shore distances using trigonometry, and so forth.

Discuss some aspects of the relationship between gifted education and special education.

Many students with gifts in specific areas and/or global giftedness also have disabilities. This complicates the circumstances of the best education programming for them. One consideration is that disabilities can prevent some gifted students from being identified as gifted. For example, if a student has visual impairment and does not receive appropriate classroom accommodations, it might not be discovered that this student also has superior language and reading abilities. Another consideration is that instruction must be differentiated in certain ways to accommodate the needs of students with disabilities, and in other ways to accommodate the needs of gifted and talented students. The challenge for educators is to combine and coordinate both types of differentiation for these doubly/multiply exceptional students. An additional aspect is that federal laws exist to guarantee education to disabled students but not to gifted students. Therefore, many U.S. state regulations for gifted education are modeled upon federal laws protecting the rights of disabled students.

Describe some factors that parents of gifted children should consider in judging the admissions quality of a gifted program or school.

Many times, schools depend on a teacher's recommendation or results of a single standardized group test to identify a student for gifted placement. Such methods typically overlook gifted students who are underachievers, have learning disabilities, and/or belong to under-represented populations such as the economically disadvantaged and racial/ethnic/linguistic/cultural minority groups. For these reasons, for years educators have advocated using multiple assessment instruments and practices. Another factor is staff development; Teachers trained in gifted education provide more effective instruction to gifted students. A school/program's teachers of gifted students should have certifications/endorsements in gifted education, and schools should provide regular in-service trainings regarding gifted students. Because gifted students are different from other students, schools also should provide guidance services to address problems such as fitting in socially with non-gifted peers and gifted hypersensitivities to the ordinary stresses of growing up and attending school. Additionally, schools should confer equivalent honors to academic talents/achievements as to athletic talents/achievements.

Describe one way in which American families with gifted children can find online information about community resources in their U.S. state.

Many websites offer information on community resources for gifted and talented students. One good resource is the website of the National Association for Gifted Children (NAGC). State associations can join NAGC as an affiliate. State affiliates then can offer NAGS Parent Affiliate Memberships to parents of gifted children. On the NAGC website, there is a Gifted by State page. As of its 2008 copy right, it linked to organizations for every U.S. state except Alaska and Wyoming. Minnesota has two gifted organizations linked. Website of state organizations for the gifted offer such information as local chapters, publications, webinars, audio recordings, upcoming events, recent news; the organization's mission statement; the state's definition of giftedness; programming standards; legislative updates; summer teacher professional development courses, workshops, institutes, conferences, etc.; e-stores; awards and scholarships; communicators; advocacy information; job postings; and more. State gifted organizations' sites also list and link to local community resources within their state including other gifted associations; college/university programs; academic/enrichment opportunities including competitions, activities, programs, and journals.

Describe one group of influences that contribute to some gifted students' underachievement in school.

Not identified as gifted earlier in school, some gifted students reach their high school years without any special educational interventions or programs designed for their unique abilities. These students may have been overlooked in the school system if their gifts exist outside of their schools' areas of curricula. While these gifted students may have the ability to perform well in certain subjects, teachers might find these students' different methods of achieving such performances unacceptable or invalid. Furthermore, such students may attend a school in which personnel lack sufficient training or experience with giftedness. By the time students receive neither acknowledgement nor support for their gifts in high school, they have felt rejected for years and are usually angry and/or depressed. Without support or appropriate curriculum they will not achieve and may drop out of school.

Describe a common learning characteristic of elementary school student gifted in math. Explain some consequences of not presenting them with challenging materials and some benefits of doing so. Give an example of a way to offer appropriate challenges.

Often children with mathematical gifts can grasp new concepts very quickly. In the general education classroom, they end up without recourse to additional intellectual exercise while they wait for the rest of the students to understand and master the new math ideas presented. Some consequences of letting gifted students go unchallenged are boredom, less than optimal thinking skills, and undesirable study habits. One way to offer them challenges is though differentiated instruction. The teacher can present math problems with graduated levels of difficulty and complexity. When students have a choice of problems at different levels, they have the opportunity to try to solve problems just within their current cognitive capacity. Such methodology challenges students with problems that are not too easy, without overly frustrating them by being out of range of their ability' it cultivates their passion for math; and it stimulates their cognitive growth and development.

Describe some typical strengths of intellectually gifted learners, and provide a potential "problem" behavior corresponding with each of these strengths.

Often intellectually gifted students are sensitive and promote empathy for others. The problem associated with this area of giftedness involvesthe students' need for the acceptance of others, as well as an accompanying oversensitivity to rejection and/or criticism. Creative and inventive students enjoy finding new ways to do things. This gift proves a boon to society in terms of innovation, but it also can be perceived as disruptive and abnormal. Intellectually gifted students often have longer attention spans than other students, can concentrate on one subject more intensively and are more persistent in the areas interesting them. Many important accomplishments are attributable to these qualities. However, these qualities also can result in the neglect of other academic subjects and/or persons and relationships, ignoring interruptions, and being thought stubborn. Many intellectually gifted students are highly alert, enthusiastic, and eager to learn; in addition, these students may possess high energy levels. These assets can be misperceived as hyperactivity/overactivity. These students can also become frustrated when inactive.

Discuss some considerations regarding academic acceleration of a gifted student in only one subject.

On one hand, many educators are more receptive to accelerating a gifted student in one subject because the student learns the remaining majority of subjects with age-level classmates. These educators often feel this type of acceleration can avoid social and/or emotional problems for the student. On the other hand, certain factors must be considered when a gifted student is accelerating in only one subject. One is continuity. If educators do not carefully coordinate the student's programming, the student could learn accelerated materials one year, only to repeat these the following year. Teachers and curriculum specialists can help by defining what is taught in each grade for each subject. Another consideration is that elementary school students might need to attend a junior high/middle school, junior high/middle school students a high school, and high school students a college course, to accelerate one subject. Transportation can present worse problems than academic or social concerns.

Cite some examples of websites on the Internet that teachers can use to find virtual lesson plans, provide students with enrichment links, and offer virtual museum tours for use in gifted education.

Once teachers have selected the content for a lesson or unit they want to deliver in a virtual (computer) environment, they can take advantage of such websites as The Educator's Reference Desk, where they can find lesson plans for any grade level or school subject. In addition to getting help creating and delivering lesson plans, teachers can incorporate hyperlinks from these site in the virtual learning environments they create for students. Students can follow these links to access virtual lessons providing content enriching the teacher's basic lesson or unit. In addition to lesson plans, teachers can find online student projects and activities, instructional materials, and virtual "field" trips. In virtual trips, students can visit otherwise inaccessible nations, museums, galleries, institutions, organizations, and corporations all over the world. Education World, Eduscapes' Digital and Virtual Museums, and the Museum of Science and Industry are examples of websites offering virtual museum tours.

Describe some categories of research wherein gifted students can conduct independent study projects, including examples.

One category of research that gifted students can use for independent study is historical research, which endeavors to reconstruct the past objectively. Some sources for gathering information include old newspapers, written records, oral history, baby books, folk songs, poems, and stories. Students could study the history of the school, the community; some aspect of the U.S. Constitution or Bill of Rights, etc. Another category is developmental research, which examines trends, change, continuity, sequences, and patterns. Developmental psychology and human growth and development are examples of developmental research areas. Students can investigate such topics as human biological development, cognitive development in children, personality development over the lifespan, etc. Another category is descriptive research, which is quantitative. Numerical statistics are depicted using graphs, charts, and tables. Students can conduct opinion or fact-finding surveys, or observational or interview studies.

Summarize some of the disadvantages of a school's using pull-out or resources services program for gifted students.

One disadvantage of pull-out programs is that school personnel may not adequately coordinate their activities with the regular classroom curriculum. Consequently some teachers - and students - wonder how some pull-out activities relate to general education subjects. Another problem occurs if resources teachers base projects on their own interests rather than students' strengths. Many teacher resources are available for activities supporting higher levels of learning, but teachers often do not avail themselves of these curriculum guides and materials. The activities they design for gifted students should not only be fun and interesting for the students, but also develop their advanced cognitive skills. Rarely, some general classroom teachers resent gifted students' weekly absence and retaliate by scheduling class parties or field trips, important tests, and/or new topic introductions on pull-out days. This unprofessional behavior uncommon but problematic. Also some busy classroom teachers do not give gifted students other differentiated learning choices for the rest of the week, mistakenly thinking the pull-out program is enough.

Describe some disadvantages of school's having self-contained classes for the gifted as a program placement option.

One disadvantage of self-contained gifted classes that continue for several years is that gifted students lack classroom interactions with non-gifted peers. One solution is to have activities such as lunch, physical education, art, and music include all students to provide interaction. Some school districts create "teams" of several classrooms, including the self-contained gifted class, so gifted and other students interact daily. Another problem occurs when either too few or too many gifted students are identified for the school's normal class size. A smaller gifted class would provoke resentment in teachers with normal, larger class sizes; a group too large would force schools to leave out some qualified gifted students. Self-contained classes also can lead to student, teacher, and parent perceptions of school elitism and isolationism. Also, though there is no proof that other students suffer, some teachers still feel that removing the gifted students leaves the other students with no role models.

Discuss some aspects of the relationship between gifted education and general education.

One factor important to consider in both gifted education and general education, since it affects both, is that compared to the numbers of special schools for gifted and talented students, there are many more public schools attended by gifted and talented students. Additionally, while some public schools include self-contained classes composed entirely of identified gifted and talented students, many more have such students mainstreamed into general education classrooms along with non-gifted students. Many public school have pull-out programs for gifted students wherein they attend resource rooms to participate in special, individualized activities with instruction from a gifted education teacher. However, pull-out programs only involve a portion of the students' school time, typically a half-day or one day at most. Therefore, it is crucial for general and gifted education teachers to coordinate their curricula and activities. Each must know what the other is doing to facilitate the student's educational progress.

Identify and define two kinds of qualitative research used to study giftedness and/or gifted education.

One kind of qualitative research is grounded theory. In this approach, a theory is developed inductively (rather than deductively, which uses the reverse process of formulating hypothesis and then collecting data to test it) from a body of data. While simpler quantitative statistical tests such as analysis of variance yield only the main effects of one variable upon another, grounded theory takes a case-oriented rather than variable-oriented approach, assuming that variables interact in more complex ways and function as a whole within the case. Grounded theory also has a comparative orientation, based on John Stuart Mill's methods of similarities and differences, to uncover the causes. Another type of qualitative research is historical. The researcher collects data about past events, and objectively evaluates these events to test his/her hypotheses about their trends, causes, and/or effects, which can help to explain current related events and predict future related ones.

Summarize the elements of four different ways of implementing a pull-out/resource room school program for gifted students.

One less common, but valuable, way to implement a pull-out program is with a multiple-year curriculum, which affords continuity. For example, using the theme of change and adaptation, fourth-graders could study environmental changes; fifth-graders could examine political changes under oligarchies, monarchies, and democracies; and sixth graders could learn of changing family roles in different regions of our country. Another method involves extending the regular classroom curriculum by studying the same subjects or topics in more depth. A third approach is basing the pull-out curriculum on national or regional enrichment programs, e.g. Odyssey of the Mind, Science Olympiad, or the Future Problem-Solving Program, and adding activities focused on the underlying reasoning and creative skills whereby students succeed in such programs. In a fourth approach, the resource-room gifted teacher decides the pull-out curriculum. In this case, the quality of program content depends largely on the knowledge, experience, and skills of the individual teacher.

Relate some of the benefits of the collaborative consultative model in gifted education.

One of the benefits of using collaborative consultative models in gifted education involves the congruence of such models with recent trends in educational reform. Another benefit affords professional development for all those participating as they contribute and share their expertise. Collaboration also allows many more ideas to be produced than any individual could generate in isolation. In addition, consultative methods make the most of the opportunity for the adult participants to use their individual differences toward achieving constructive goals. Another advantage of the collaborative consultative model is that it is amenable to the school administrator's taking the role of facilitator. Moreover, using collaboration and consultation between teachers, parents, and others from outside of the school affords the benefits of parents' satisfaction through their direct involvement, contributions, learning from others, and seeing their children's knowledge and social skills improve.

Explain cluster grouping in educational settings, its use with gifted students, and some of its pros and cons.

One practice that schools employ to meet the educational needs of their gifted students is to use cluster grouping. This practice entails grouping all of the gifted children at one grade level inclusively in one regular classroom. For example, if a school has three separate fourth-grade classrooms and a total of six fourth-graders identified as gifted, these students would all be placed in one of those classrooms rather than being distributed among several fourth-grade classrooms. This works for students gifted in individual subjects; a student with verbal gifts can be in a different classroom than a student with mathematical gifts. Some benefits are that cluster grouping is inexpensive and that students can move fluidly in and out of cluster groups. A drawback of cluster grouping is that globally gifted students cannot attend multiple classrooms simultaneously to meet their advanced needs in multiple subjects. Another consideration is that teachers must differentiate their instruction for greater variations in ability levels within one class.

Give some examples of enrichment activities for young gifted children that develop skills in making inferences, and in math and reading/reading readiness/language skills.

One way young gifted children can be encouraged to exercise their cognitive abilities is through inferences, which require them to move beyond literal and concrete perceptions to more abstract thinking involving speculation based on existing information. For example, you can show a child a picture of people wearing uniforms and ask what their jobs may be, or show a picture of people with various facial expressions and ask what they might be feeling. In both cases, ask what could have happened before and after what is in the picture. For children with mathematical gifts, it is important to incorporate fun, game-like math activities into daily living routines. For developing reading readiness skills in young gifted children, you can label common household objects with their printed names on 3x5 cards. Start open-ended stories; let them make predictions and hypotheses about what will happen next and what will happen if that occurs.

Identify, define, and give examples of four kinds of academic vocabulary commonly used in social studies courses that gifted students should know and use.

Originally identified by Deborah J Short (1994), social studies employs four kinds of academic terminology: instructional/directional, concrete, conceptual, and functional. Instructional or directional tools like maps use terms such as north, south, east, west, above, below, etc. Concrete terminology refers to factual information; for example, names of laws like the Stamp Act, of events like the Industrial Revolution, and/or proper names of historical figures; dates or years of historical events, etc. Conceptual vocabulary includes words referring to ideas, such as "democracy," "taxation," "representations," "Enlightenment," "utilitarianism," "supply and demand," etc. Functional vocabulary consists of terms denoting certain academic/cognitive functions or processes, such as "sequencing" events in chronological order. In addition to making sure they know about these kinds of academic vocabularies, teachers should encourage students to use terms from each of these domains during classroom discussions and in their group projects, oral reports, and presentations.

Give examples of ways parents can nurture their children's giftedness.

Parents can begin reading to children regularly as soon as the children are able to attend. Furthermore, as children develop, parents can encourage children to read to them as well. As children grow, parents can observe and encourage their interests/aptitudes. Providing books for verbally precocious children is crucial (libraries offer a good option if funds are limited), since good writers begin as good readers. Parents can give children with mathematical talents numerical puzzles and games. For children interested in producing reactions by mixing substances, junior chemistry sets or household materials can be provided. Beginner telescopes and trips to the local planetarium are good for children fascinated with stars and planets. Parents can provide interested children with introductory geology sample sets and take them on local field trips to discover rocks, minerals, and/or fossils. Basic art supplies are important for children with artistic abilities. Parents should allow and encourage free expression, exploration, discovery, learning, skills development, and new insights.

Indicate the status of parental access to children's school records. Give an example of where different school records on a gifted student may be found. Identify common misconceptions parents may have about IEP planning meetings they are to attend.

Parents have the right to request and receive copies of all of their children't school records, including the results of any tests. These may not all be in one place. For example, the guidance counselor might maintain records of students' IEPs and educational programs, while IQ and other test results would be maintained by the school psychologist. Gifted advocates advise that parents of gifted students should obtain copies of current tests used to identify their child as gifted and of tests used to determine the students' achievement levels. Common misconceptions among all parents, including those with gifted children, is that the school IEP team has already created the IEP, that it cannot be altered, and that they are attending the meeting only to review and approve/disapprove its content. As such, parents must be informed of their roles as equal team members in contributing to, developing, and/or monitoring their child's gifted IEP.

Describe some factors that parents of gifted children should consider in judging the quality of a gifted program or school.

Parents should be able to identify the philosophy and goals of a gifted program and/or school. Because giftedness is a lifelong trait, gifted students' academic needs will have some continuity. If program goals differ by age, these differences should reflect instructional variation, which should be both age-appropriate and appropriate for gifted students. Gifted programs should include both acceleration, i.e. offering instruction at a faster pace for suitable for gifted students; and enrichment, i.e. extending the curriculum by increasing the depth of study of a subject appropriately to gifted students' abilities and needs Another essential factor involves offering multiple options in gifted education. Students gifted in the language arts are different from those gifted in math; mildly gifted and profoundly gifted students differ from one another. Owing to individual differences such as personalities and learning styles, even students with similar levels of giftedness in the same subjects can be very different. As a result, it is critical to include multiple ways to address these multiple educational needs.

Identify some skills that gifted students can develop through engaging in a creative problem-solving process.

Participating in creative problem-solving develops the higher-order cognitive skills of gifted students, which helps them to apply their knowledge for solving problems. According to experts, there is more emphasis in this process on learning "how to think" than on acquiring information. Creative problem-solving helps gifted students enhance their analytical skills as well as their creative abilities. It builds their skills in spoken and written communication and encourages gifted students to develop skills for conducting research and improving these skills. When gifted students participate in groups to solve problems creatively as a team, the process contributes to their developing responsibility as group members. The creative problem-solving process provides gifted students with guidance that enables them to develop not only responsibility, but also more self-direction. This process also furnishes gifted students with a model for problem-solving they can incorporate into their lives. Moreover, it engages gifted students' interest in the future.

Give examples of the beliefs and goals of educators and advocates of gifted students regarding the role of the visual and performing arts in education and in life.

Persons and groups dedicated to the education of gifted students believe that the visual (drawing, painting, sculpture, photography, cinematography, etc.) and performing (singing, dancing, acting, etc.) arts are essential parts of healthy, productive lives. The Arts Network of the National Association for Gifted Children, for example, finds that "...the health and productivity of a society is reflective of the degree of artistic expression among its citizens." This network thus commits to the initiation, development, and implementation of teaching resources and strategies to support students aptitudes and interests in these areas, including furthering professional and public acknowledgement of the visual and performing arts as a necessary domain of giftedness, promoting more research into the topic of artistic gifts and talents, furnishing practical teaching methods and resources to cultivate artistic perception, appreciation, values, and expression.

Contrast the qualitative and quantitative methods of research of giftedness and gifted education in terms of their respective natures and characteristics. Discuss which circumstances indicate the use of each type of methodology.

Popular conceptions of research methodologies characterize quantitative methods as using hard data, numbers, objective information, and finding that can be generalized to the larger populations represented by study samples; and qualitative methods as using "soft" data, words, subjective impressions, and finding more specific to the study's subjects but not generalizable to larger groups. Each method has its own strengths and weaknesses. Rather than choosing one or another method by personal preference, many researchers (cf. Neill, 2006), believe the nature of the research question should dictate the most applicable methodology. Neill also points out that qualitative and quantitative methods are not opposing paradigms; rather, each has its use, and the existence of mixed-methods studies proves they can be used together. Researchers are more likely to use qualitative methods earlier in a study to learn more comprehensively about a topic, using quantitative methods later on to seek precise answers to more specific questions.

Discuss some issues related to timely homework completion by gifted students when they have NOT been given double assignments, including how to address these in the gifted IEP (GIEP)

Problems result when gifted students are given both general and gifted education homework, thus doubling their load. When such doubling is not the case, some gifted students still can have time management issues with homework. Here are some factors to consider: many gifted students are perfectionists. As such, they can expend excessive, unnecessary time trying to perfect their homework. Also, while some gifted students process information faster than non-gifted students, many do not. In fact, researchers have found that measures of working memory and processing speed on standardized IQ tests are poor indicators of giftedness. Timely homework completion is a skill that develops over time. If problems have developed over time, such problems also are resolved over time. These problems are addressed in the Specially Designed Instruction section of the GIEP. While teachers or local school districts may require homework completion to pass a course, many State Departments of Education do not.

Describe the Guilford Battery of tests for creative thinking developed by J.P. Guilford in the 1960s.

Psychologist Joy Paul Guilford (1897-1987) formulated his Structure of the Intellect model in 1962, identifying 180 different kinds of thinking, many of them divergent. He found that the types of divergent thinking most pertinent to creativity were the abilities to generate new information from existing information and to transform knowledge or experience into new patterns and configurations. Based on this model, Guilford developed a battery of tests for creative thought. This battery's ten tests each measure divergence in producing the following: 1 Names for Stories - semantic units 2 What to Do with It - semantic categories 3 Similar Meanings - semantic relationships 4 Writing Sentences - semantic systems 5 Kinds of People - semantic implications 6 Make Something Out of It - figural units 7 Different Letter Groups - figural categories 8 Making Objects - figural systems 9 Hidden Letters - figural transformations 10 Adding Decorations - figural implications

Summarize some of the advantages of a school's using a pull-out or resource services program for gifted students.

Pull-out programs give gifted students the benefits of weekly classroom interactions with their intellectual peers and of sharing experiences involving their individual strengths. Many gifted students look forward to this so much that they will never miss school on their pull-out day. Pull-out classes often have open-ended enough curricula to allow freedom of creative exploration, enlisting the teachers' and students' skills and imaginations to design projects of interest and value, which is another advantage. Having gifted resource-room teachers is also financially advantageous to school districts, as these teachers usually work with all gifted students at various grade levels, teach lessons in regular classrooms, conduct in-service trainings for other teachers and staff, field parents' phone calls as delegated by their school principals, and perform other functions entailing intensive communications with varied audiences.

Summarize the main features of quantitative research methodologies in terms of their contrast with qualitative research methodologies, and provide an example of how they pertain to research of giftedness/gifted education.

Qualitative methodologies tend to be inductive, i.e. they proceed from the specific to the general and generate new theories. Quantitative methods tend to be deductive, i.e. they proceed from the general to the specific and test existing theories. Qualitative methods incorporate more subjective aspects, while quantitative methods restrict finding to objective facts and figures. For example, if you wanted to find out which of two curricula is preferred by individual gifted students in a general classroom, you likely would use qualitative methodology and ask students which curricula they like better - thus introducing a subjective element. If you wanted to find out which curriculum resulted in higher scores on standardized achievement tests taken by the gifted students in the general classroom, you likely would use quantitative methodology to determine differences in test scores and to discover whether a correlational or causal relationship (or both) exists between the curricula and the test scores.

Identify and describe three general methods for collecting and analyzing data used in qualitative research which can be used in studying giftedness and/or gifted education.

Qualitative research is descriptive, experiential, and interpretive rather than statistical or numerical, as a quantitative research. Qualitative research demands rich, detailed descriptions to reveal the research process and to communicate implicit knowledge. As such, qualitative research uses three methods of data collection and analysis: observations, interviews, and self-reports. In observation, the researcher observes the nonverbal and verbal behaviors of study subjects (avoiding influencing the subjects' behavior insofar as is possible) and records descriptions of observed behaviors. Interviews are interactive with the subjects: the researchers first ask the participants to describe specified events or phenomena orally, and then the researcher records the participants' responses. In Self-Reports, researchers also may ask participants to write down descriptions of their experiences. Researchers analyze the initial data they collect to inform and direct further data collection. All these methods can be used to obtain data from gifted students, their parents, teachers, non-gifted peers, etc.

Characterize qualitative research of giftedness and gifted education and how it contrasts with quantitative research.

Quantitative research methods use statistical procedures to measure or quantify phenomena in giftedness and gifted education, while qualitative research methods do not use such measures. Qualitative research is interpretive in nature. Implicit in the qualitative approach is the assumption that one must investigate giftedness as a whole in order to understand it. Quantitative research typically analyzes components of giftedness/gifted education. Proponents of qualitative research argue that such piecemeal examinations overlook important parts of the holistic appreciation they advocate. Rather than an absolute reality posited in quantitative research, qualitative methods propose multiple realities, which themselves vary across different times and places. Holistic understanding of the complexity, richness, and depth of giftedness/gifted education often reveals meanings that help researchers gain new insights. While all quantitative research is conducted according to an overall structure, qualitative research methods vary in their individual theoretical orientations relative to the specific topic or phenomenon under study.

Explain how teachers of gifted students can use Bloom's Taxonomy as a reading strategy to enhance content literacy in social studies. Identify and define the skills organized by Bloom's Taxonomy.

Recent research finds a student's academic vocabulary in a subject as most predictive of their success in learning the content of that subject. For literacy in a content area, students must construct meaning from what they read. Using linguistic strategies assists students in finding meaning from interacting with their texts by interpreting, organizing, and retrieving the information they encounter. One way that teachers can help students to do this is by using Bloom's Taxonomy when writing questions for study, discussion, review, and/or testing; and when designing projects or activities for gifted students. Bloom identifies the following skills, in order of increasing complexity; knowledge, or knowing facts such as names, dates, events, etc.; comprehension, or understanding the facts known; application, or applying understanding to other situations/examples; analysis, or breaking information down into its components; synthesis, or creatively combining/integrating components; and evaluation, or assessing the accuracy, meaning, relevance, utility, and application of information.

Identify some gifted education and IEP issues that many parents of gifted students commonly encounter across public school districts.

Recurring issues that many parents of gifted students in public school districts have reported include the following: 1. when invited to IEP meetings, many parents report they were not informed of their status as equal partners in IEP planning 2. not informed that IEP meetings are working meetings 3. not informed that parents should bring their own ideas to the meeting 4. many parents report they were not informed that accommodations to general classrooms could be written in gifted IEPs If parents find their school has not assessed a student's present level of educational performance.... 5. many express that they were not informed how to request testing or that assessment results could be used to develop their child's gifted IEP. Parents often state that 6. they are told that acceleration in grade or subject violates the school district's policy 7. that their gifted program is an "enrichment only" program 8. districts often indicate their pull-out program as their entire gifted program

Discuss some general mental health considerations for educators and other adults who work with gifted students.

Research finds gifted students no more prone to experience mental health problems than are other students. Nonetheless, if educators and counselors have limited knowledge and experience of gifted students, their perceptions can be influenced by exclusively positive stereotypes of the gifted. Such a perception can be dangerous, in that adults may not recognize, and hence not address, counseling needs and/or developmental issues in gifted students. Some gifted characteristics can become liabilities in this sense. Also, adults unfamiliar with gifted students' specific characteristics may incorrectly interpret some divergent behaviors as pathological symptoms. Another consideration is that gifted students, wanting to preserve their competent reputations and avoid disappointing invested adults, often do not request needed assistance. Adults who are both uninformed regarding the complexity of gifted students' concerns and also dazzled by the gifted students' abilities are less objective, can overlook students' vulnerabilities, and cannot work effectively with them. Adult respect for gifted students is more appropriate than awe. As in curriculum and instruction, differentiation is counseling services is indicated for the gifted.

Give examples of some benefits to school districts for participating in research studies regarding effective methods in gifted education.

Research studies regarding effective teaching methods in gifted education often develop curriculum units to test in actual practice at schools. Such units are designed to challenge gifted students, differentiated for all student levels in general education classrooms and based upon national standards. Such units educationally benefit more students at more levels than non-differentiated instruction. Participating school districts often receive tools for the identification and the assessment of gifted students free of charge, which represents a financial benefit. Major research centers also provide support for educators' professional development, benefiting school districts financially and benefiting teachers, other school personnel, and students. This support often includes online technical assistance, an additional benefit addressing the increasing use of online resources and tools. Participation in a research study conducted by a center offering professional development courses for educators also gives priority to some members of school district staffs in registering for these courses.

Give some examples that show hos a "scavenger hunt" can help gifted students not only comprehend social studies reading literally but also approach it as historians using critical thinking skills.

Researchers (Doty, Cameron, and Barton, 2003) indicate that "...teaching reading in social studies is not so much about teaching students basic reading skills as it is about teaching students how to use reading as a tool for thinking and learning." One way teachers can help gifted students construct meaning from text is a "scavenger hunt" through the book. For example: ask them how many chapters and/or sections the book has and how it is organized. Ask what kind of material is at the beginning of the text and the significance of this. Ask them what kinds of skills or techniques the reader may require to read and understand the text. Ask them to identify special features in the student textbook that do not appear in trade books, and how these features can help them to understand and organize the text content. Finally, ask them how their answers to the preceding questions will help them to read the text better, and why.

Name and explain five environmental constraints researchers have found that tend to stifle creativity in gifted students.

Researchers (e.g. Amabile, 1983a, 1996; Hennessey, 1996) have identified five environmental constraints that destroy internal motivation and creativity: 1 - Expected Reward - Students work to receive some externally bestowed reinforcement, rather than being internally motivated to be creative for its enjoyment. 2 - Expected Evaluation - students limit creative exploration, working instead to score well on a test. 3 - Competition - rather than exploring a subject creatively to see where it leads, students focus on besting other students by obtaining the highest scores. Such competition limits creativity, since students' pursuits are narrowed to traditional academic tasks and to achieving the best among typical student responses rather than anything original or different. 4 - Surveillance - when students know they are being observed, they feel less free to engage in creative exploration or find divergent solutions. They are more likely to conform and less likely to be creative. 5 - Time limits - when students know they are being timed, they feel pressured. Finishing on time interferes with the additional learning and production afforded by creativity.

Summarize some issues associated with using the Full-Scale IQ (FSIQ) score on the Wechsler Intelligence Scales for Children, 4th edition (WISC-IV, 2003) to identify giftedness.

Researchers at the Gifted Development Center observe that since the WISC-IV includes more emphasis on processing skills than earlier versions, this confounds the identification of giftedness because gifted students tend to score higher on abstract thinking measures but lower on processing measures. This characteristic reflects the asynchronous development typical of gifted children. Averaging these measures lowers the FSIQ score. The researchers find abstract thought a better index of intelligence than processing speed or working memory. They also point out that many educators mistakenly assume gifted students are naturally quicker processors. Some are, but many others are perfectionists and/or reflective thinkers, which slows down processing time. Also, many gifted students perform well on meaningful tasks, but not on tests of short-term memory containing material with no meaning for them. Flanagan and Kaufman (2004) recommend not reporting the FSIQ score if the difference between the highest and lowest Composite score is 23 points or more.

Summarize which subscales of Wechsler Intelligences Scales for Children, 4th edition (WISC-IV, 2003) are recommended for identifying giftedness, which additional measures can be used with the WISC-IV instead of its Full Scale IQ (FSIQ) score, and some advantages of these.

Researchers at the Gifted Development Center recommend, often subscales required by the WISC-IV, using only the following six to identify giftedness: 1. Vocabulary 2. Similarities 3. Comprehension 4. Matrix Reasoning 5. Picture Concepts 6. Block Design The first 3 subscales comprise the Verbal Comprehension Index; the last 3 comprise the Perceptual Reasoning Index. The researchers find the two indices most relevant to measuring giftedness. The General Ability Index (GAI) is recommended by Flanagan and Kaufman (2004) as an additional measure, computed from the six subscales/two indices named. Dumont and Willis provided the Dumont-Willis Indices (DWI); the DWI-1 is similar to the GAI, computed from the Verbal Comprehension and Perceptual Reasoning indices. The DWI-2 averages Working Memory and Processing Speed. They advise only computing both DWI-1 and DWI-2 for gifted students if scores are similar. Advantages include greater efficiency, time- and cost-effectiveness, and accuracy without Working Memory and Processing Speed scores, which confound results for gifted students.

Describe some of the characteristics of the profile of a "double-labeled" gifted student.

Researchers identify "double-labeled" gifted students as those with both giftedness and physical, emotional, or learning disabilities. Although dual diagnosis is possible with thorough evaluation, most educational programs for the gifted neither identify such students nor afford them differentiated instructional programming that integrates and meets their specialized needs. Because of their combinations of gifts with disabilities, schools can frequently overlook these students, as many of them display behaviors that school personnel do not associate with giftedness. Students with learning disabilities can display poor organizational skills, problems with memory, difficulties with reading, poor handwriting, and etc. Those students with emotional or behavior disorders may exhibit disruptive behavior. Due to the contradictions of their abilities and disabilities, these students are often confused and stressed and suffer from low self-esteem. Often these students are either perceived as average and therefore ignored, or targeted for remediation of their disabilities while their respective gifts are overlooked.

Compare the areas of verbal comprehension and perceptual reasoning with the areas of working memory and processing speed as measured on a number of IQ tests in terms of the degree to which to indicate giftedness. State the implications of full-scale IQ scores that average the former tow areas with the latter two areas of gifted identification.

Researchers compared Wechsler Intelligence Scale for Children (WISC-IV) scores of a gifted student group with scores of a control group from the WISC-IV's normative sample. The normative sample had little IQ variation - only 4 points between highest and lowest subscale scares. Working Memory and Processing Speed scores were not lower enough than Verbal Comprehension and Perceptual Reasoning scores to lower their Full-Scale IQ (FSIQ) averages. In contrast, the gifted sample averaged 27.4 points difference between highest and lowest subscale scores; the greatest variation was 69 points - over four standard deviations. Almost 60% of gifted students had 23-point differences between Verbal Comprehension and Processing Speed Composites. Gifted students scored 25 points higher than the norm group in Verbal Comprehension, but differed by below 2 points in Processing Speed. This trend implies gifted students are superior in verbal abstract reasoning but not processing; as a result, the WISC-IV's FSIQ should not be used for gifted identification.

Discuss some aspects of creativity that measurement instruments can identify and some that such instruments do not predict.

Researchers find that many instruments designed to measure creativity can identify the characteristics of fluency in ideation and divergence in though, commonly accepted as components of creativity. However, they find that these measures do not predict future creative behaviors in a student identified as creative. Some scientists attribute this characteristic not to psychometrics but poor methodology - e.g. some studies were not continued long enough, data not having normal distributions did not receive adequate statistical procedures, or the criteria for outcomes in longitudinal studies were not well operationalized. Some researchers also note that while personal definitions and theories of creativity are not included, they should be. These researchers have proposed that individuals acting creatively are directed by their personal beliefs about the nature of creativity, how to nurture and measure creativity, and that these beliefs may differ substantially from the theories that experts in the study of creativity have developed.

Discuss some characteristics of gifted students relative to achievement motivation. Explain why traditional teaching methods are less motivational to gifted students.

Researchers have discovered that when gifted students consistently bring high skill levels to schoolwork, externally imposed limitations that represents threats to their senses of self-determination are particularly likely to affect them negatively. Gifted children frequently know they have the ability to exceed their nongifted peers and performance; therefore they do not rely as much on external rewards, the contingencies for earning them, or feedback from the teachers. And more importantly for them involves preserving their internal motivation, which cannot be forced or learned, but can too easily be washed. While research finds get to students typically highly motivated, it also find that motivation in classrooms is problematic for them. This finding is attributable to get to students not responding well to rigidly structured, specific task or assignment, which not get to do that often. It's her self motivated instead of teacher motivated, get to do is perform better when given choices, flexibility, and unstructured tasks.

Describe some of the characteristics of the profile of a "successful" gifted student.

Researchers have estimated that about 90 percent of students identified as gifted fit the profile described as "successful". These children are most often identified as gifted because they have learned which behaviors adults prefer; as a result of this learning, these children demonstrate the adults' preferred behaviors. They typically achieve high scores on IQ, aptitude, and achievement tests. They are not likely to display problem behaviors because they crave adults' approval of and know which behaviors will obtain this desired result. Although many people expect such children to succeed independently, such success is often not the case. The researchers find these children tend to depend on adults for direction. Bored with regular curriculum, these children often "work the system" in order to achieve with minimal effort, not developing the autonomy necessary to pursue their own interests and gifts. Liked by peers and rewarded by adults, these students grow up socially well-adjusted but not prepared for life's changing demands; as adults, they may be underachievers.

Relate some factors in schools that contribute to underachievement among gifted students from minority backgrounds.

Researchers have found that among gifted black students, for example, underachieving students have reported experiencing less positive teacher-student interactions, less supportive classroom climates, insufficient time to understand lesson material, and feeling uninterested and unmotivated toward school. They also attributed part of their lack of interest to their schools' lack a multicultural education. Teachers often have lowered expectations of minority and low-income students, especially for teachers with inadequate preparation in both gifted education and multicultural education. Such teachers are less likely to refer minority students for gifted education services, and with lower expectations they are also less likely to identify these students as underachievers. Without access to suitable educational services that address their individual strengths and needs, minority gifted students can become frustrated, bored, lose interest in school, and become underachievers.

Discuss some general aspects of individual and group differences among gifted children. Summarize two theoretical models of group differences among gifted students.

Researchers have pointed out that such individual factors as genetic composition, life experiences, personality, and personal development, as well as the influences of families, interpersonal relationships, and educational experiences result in individual variations in the ways in which gifted children respond to and express their special abilities. While the research has acknowledged individual differences among gifted children for at least 50 years, less work has been done to identify various groups among the gifted. Roeper (1982) theorized five types of gifted children based on their approaches for coping with and expressing their feelings. These types include: 1. the Perfectionist 2. the Child/Adult 3. the Winner of the Competition 4. the Self-Critic 5. The Well-Integrated Child. Betts and Neihart (1988) identified six "profiles" of gifted students based on their needs, feelings, and behaviors: 1. The Successful 2. The Challenging 3. The Underground 4. The Dropouts 5. The Double-Labeled 6. The Autonomous Learner

Describe some of the characteristics of the profile of a "dropout" gifted student.

Researchers identify a salient characteristic of the "dropout" gifted student as anger. These students have experienced long-term failure of the system to meet their needs. Disappointment with themselves and with adults along with their feelings of rejection can lead to depression. In these students, this depression may manifest as social withdrawal or as defensive acting-out behavior. These students often have abilities and interests in unusual areas not included in school curricula, so they are not affirmed or rewarded for these abilities and interests. School may seem irrelevant or even antagonistic to those students with very different gifts. Many dropout students were not identified as gifted until later, e.g. in high school. This long-term lack of recognition and appropriate educational programming engenders resentment, bitterness, and low self-esteem. Educational experts recommend that these students need individual and group counseling, a close working relationship with a trusted adult, and diagnostic testing for possible remedial areas. Experts also state that traditional programming is not indicated for "dropout" gifted students.

Summarize some recent issues and trends in gifted education and their implications.

Researches in gifted education have identified issues in some gifted programs that reflect common concerns in gifted education. Programs including part-time pull-outs, cluster grouping within general education classrooms, and center-based gifted programs all share some common concerns. Even with well-established, exemplary curriculum units, a needs exists to reconsider the organization and focus relative to progress in cognitive learning theory and to content standards. The emphasis on documenting how instruction supports students' learning requires educator to consider differentiated instruction in greater depth. Performance-based assessments have gained greater prominence in evaluating learning because they are more congruent with higher-order content standards such as enabling more student demonstration of reasoning and problem-solving skills in specific subjects. Implications include that assessment must play a more central role in program and curriculum planning rather than depending only on traditional standardized achievement tests. Furthermore, collection and reporting of performance-based student impact data as well as more integration of assessment techniques and economy in analyzing data are indicated to promote both learning and accountability.

Describe a program placement option for gifted students consisting of a self-contained "special education" program, and some of its advantages.

Self-contained gifted classes consist entirely of gifted students. In districts having these, they are at the elementary school level. If middle and high school students are placed in each subject via ability grouping, these classes may function as self-contained. However, true self-contained gifted classes for all subjects do not exist in regular public secondary schools due to subject departmentalization. Advantages include the ability to design more gifted-appropriate instruction overall; higher gifted student achievement levels than those in pull-out gifted programs; usually no extra expense, as the gifted class will have the same number of students as a regular class at that grade level; the greater ability of gifted students to relate to their intellectual peers; and a more supportive classroom atmosphere, more conducive to freedom of expression and more encouraging relative to individual student strengths and weaknesses, which allows gifted students to perform better and more consistently.

Give a description of differentiated homework as one form of accommodation to provide specially designed instruction for gifted students within general education settings, including examples.

Since gifted students most often are placed in general education classrooms and given supplementary gifted services, they could be assigned the same homework as the regular class, plus another set of homework from their gifted program. Thus gifted students can end up feeling punished rather than rewarded by having to complete a double amount of homework. Differentiated homework can avoid this problem by substituting homework more appropriate to the student's gifts instead of the regular class homework rather than in addition to it. Differentiated homework also can eliminate excessive rote work/repetition beyond the gifted student's needs. For example, assigning math puzzles instead of memorization lets gifted students apply their knowledge. Assigning a gifted student an additional vocabulary list, with homework only on the extra list, and/or having a gifted student write two longer stories per week instead of five shorter stories are other examples.

Explain the importance of instruction connecting math and science to real life for gifted students, and provide some examples that teachers can use.

Since math and science are so often heavy in calculations and facts, students are usually not taught how these subjects relate to the real world. All students, especially gifted students, need to find meaning in an academic subject and understand how it applies in life. We may not appreciate enough that math and science are not like people whose opinions we can disagree with; they provide hard, objective, unchangeable facts. Teachers can show students the consequences of ignoring facts with examples like these: mathematicians and engineers advised not launching the Challenger space shuttle, but management overruled them, thereby leading to the deadly explosion. Pop singer Aaliya died in a plane crash after pilot and crew ignored the mathematics indicating airplane overload and flew regardless. A mathematician proved racial bias in jury selection by calculating that the probability of fair selection was approximately 1 in 1,000,000,000,000,000.

Comment on individual differences within the population of gifted students compared with variation in the general student population, as well as the social relevance of this comparison.

Social contexts such as geographic residence, socioeconomic status, and cultural background affect personal, family, and community definitions of giftedness. Some groups place high value on academic success. Other groups place higher value on creativity, while yet other groups place higher value on adaptability, i.e. ability to adjust to changing circumstances, including fitting into a social group, more than excellence. Finally, some cultures place priority on the value of service to others. Multiple types of giftedness exist besides intellectual ability. Even within that one type, emotional and social factors are significant considerations. The range of measured IQs among gifted students is as large as the range in general education classrooms. Difference between a moderately gifted child and an extremely gifted child can mirror differences between a child of low-average ability and a moderately gifted child. Psychologically and socially, variation is as great among gifted students as between gifted students and others. Increased variation can increase social problems. Nonetheless, because all gifted students vary in social adjustment, educators are also cautioned against making assumptions about social issues.

Describe some general steps recommended to make school environments more receptive and conducive to protecting intrinsic motivation and creative performance in gifted students and others.

Social psychologists and other educational researchers have amassed a body of literature over 30 years regarding intrinsic motivation, creative performance, and how to promote these attributes in the classroom. Research increasingly has produced evidence that educational reforms to make our schools more conducive to internal motivation and creativity can benefit all students, not merely gifted students. Some steps that have been suggested (Hennessey, 2005) to accomplish such reforms include the following: Teachers need to make every effort to create atmospheres in their classrooms wherein students feel in control of their learning processes. Teachers and school administrators need to attain some distance and perspective in order to make objective, critical reviews of their schools' current reward systems and the types of incentives used. In addition, when external rewards are being used, educators need to help students achieve distance from the limitations these impose. Educators must help students develop proficiency in knowing their own weaknesses and strengths.

Describe some enrichment skills in interpersonal interactions and group relationships that gifted students in social studies courses should develop.

Social studies skills for gifted students that fall under the area of interpersonal and group relations skills include the following: students should learn to define social studies terms and identify fundamental assumptions made in social studies. They should learn how to identify the existence of conflicts between or among the values of the people and groups involved in social studies topics and issues. They should learn to recognize when and where stereotypes exist and to avoid being influenced by them. In addition, they also should learn how to avoid engaging in stereotypes, themselves. They should become able to acknowledge various points of view. Gifted students in social studies courses should develop empathy for and understanding of others. They should participate in group discussions and planning in their social studies classes. Finally, they should be able to collaborate in order to attain goals cooperatively and take responsibility for completing tasks.

Discuss some teaching strategies related to using vocabulary, associations, and questions for teaching enriched social studies to gifted students.

Social studies teachers can help their gifted students prepare for a reading assignment by giving them a chance to preview the academic vocabulary or terminology used in the text. For example, the teacher can use a "word splash," a collection of key terms used in a section of text arrayed on a page with the most central concept or term in the center and related terms around it. The class can discuss these terms for comprehension before reading and for recognition during reading, and the teacher can post them on the wall or board. To make associations or connections, teachers should have students ask themselves what they think they know about the subject before reading. This achieves the dual purposes of uncovering student biases and/or misperceptions, establishing a context for the reading and making them feel familiar with the subject, and stimulating their engagement and interaction with the material.

Explain how teachers can help gifted students in enriched social studies discriminate between sequential and causal relationships in texts they are reading. Explain how graphic organizers and paraphrasing can help students as post-reading strategies for review and synthesis.

Social studies teachers can point out to their gifted students that when authors of reading materials use terms like "...and then...", "next," subsequently," thereafter," "later," and "finally" to represent a string of events, this indicates a chronological sequence, but it might or might not also indicate a cause-and-effect relationship. Teachers should advise their students to seek other clues in the writing before they assume that one event or action actually caused the next one. After reading, students still may need support to figure out the author's primary message or argument. Graphic organizers such as concept maps, flow charts, or outlines help students to visualize verbalized ideas. Teachers can have students paraphrase what they read in 3-5 written or spoken sentences to show their comprehension. Paraphrases should include the subject/topic, the main idea, key terms, and the most important details.

Explain some of the complications for students with both giftedness and a co-existing disability.

Some gifted students also have a co-existing physical, emotional, psychiatric, behavioral, or learning disability. This co-existing occurrence of giftedness and disability complicates diagnosis. For example, intellectually gifted children can exhibit a number of behaviors that appear very similar to the behaviors of children with attention deficit hyperactivity disorder (behaviors such as high energy and activity levels, impulsive actions, rapid speech, etc). They may exhibit compulsive organizing and possible obsessive-compulsive disorder. Most educational programs for gifted students do not address concomitant disabilities, while conversely most programs for disabled student do not address giftedness. Researchers are endeavoring to remedy these errors. At times teachers might overlook these students' areas of giftedness and instead focus on the behaviors produced by the students' disabilities. Teachers may not consider that a child with messy handwriting or disruptive classroom behavior, for example, might exhibit giftedness in areas such as art, science, music, dance, or athletics.

Describe one group of factors that can contribute to underachievement in some gifted students.

Some gifted students express their gifts during the earlier parts of their childhood by pursuing areas of special interest to them, excelling in certain school subjects, and working on advanced projects. Some students suddenly drop all these activities, deny their previous interests, or claim they have lost interest or outgrown them. Experts have found girls most likely to exhibit this trend during middle school/junior high school, while boys are more likely to exhibit this trend in high school. Researchers attribute the change to social and emotional needs, especially in school. Girls approaching and entering adolescence experience a great increase in their needs for peer acceptance and for a sense of belonging. Consequently, some gifted girls will hide or deny their giftedness in order to fit in with a non-gifted group of peers. Boys in high school often experience pressure to play sports, a celebrated skill, while academic excellence without athleticism is ridiculed.

Describe one set of factors that can contribute to underachievement in some gifted students.

Some gifted students figure out early on which behaviors are desired and rewarded by adults, and then these students use their superior abilities to produce these behaviors. They are successful according to adult standards and appear to have high self-esteem because of the adult reinforcement they receive. They are not identified with behavior problems because they comply with adults' wishes in order to succeed. However, such students, especially when placed in regular classes with mainstream curricula, often secretly feel bored with school. These students use their expertise to get through school by expending the minimal effort possible. As a result, they do not develop independence and skills needed for self-direction, nor do they explore subjects of personal interest to them. Instead, they rely on adults for direction, guidance, and feedback. They often develop into adults who display competence but not originality or imagination. When they continue these behaviors in adulthood, they can become underachievers in relation to their ability.

Describe one set of variables that can lead to underachievement in some gifted students.

Some gifted students have superior abilities that they can express congruently within the traditionally accepted framework of academic performance and achievement. Other gifted students have superior abilities best expressed outside of this framework. These students often think in more markedly different ways than other students, who often fail to understand them. They may also have talents not included in school curricula. They do not receive rewards for their gifts in the same manner as students with more traditionally acceptable gifts. As a result, the gifted students become frustrated. Due to the unconventional nature of these students' gifts, educators often do not identify them as gifted, so they receive little instructional support. They also are often highly creative. Their spontaneity can be perceived as disruptive in school. With their self-esteem compromised and their gifts not nurtured, such students likely underachieve in school and exhibit increased risk of dropout if they do not receive specialized instructional attention.

Relate some guidelines for educators in conducting program evaluations of their services to gifted and talented students.

Some of the guidelines given by the National Research Center on the Gifted and Talented for program evaluation involve the following. In program development, educators should incorporate evaluation procedures into the earliest planning stages. They should develop a specific plan for how they will use their evaluation results. They should use multiple sources such as students, teachers, parents, administrators, and school board members, for information to develop clear program goals and descriptions. They should allow enough time and funds for evaluations. They must also prepare/train their school staff to conduct assessments and analyze the results. Educators should identify clearly all parties needing, and/or interested in, evaluation findings, and engage them in the process of evaluation. Because the results of gifted education programs are complex, educators must find or create appropriate assessment instruments. Data collection methods should reflect the organization and objectives of gifted programs, e.g. portfolio assessments, out-of-level testing, product ratings with inter-rater reliability, etc. Educators should report evaluation finding timely to all those involved, including follow-up recommendations.

Discuss some common characteristics and needs of intellectually gifted students that instructional curriculum design and delivery must address.

Some qualities that educators have found intellectually gifted students to exhibit include curiosity and an inquisitive nature (i.e. wanting to find out and know things, to understand processes, and to answer questions); complexity in their thinking, personality, and behavior; possessing an extraordinary capacity for learning; more diversity among their interests in learning; a preference for learning subjects holistically rather than in parts or analytically; a tendency to learn more intuitively than methodically or logically; tendencies toward perfectionism; and a corresponding tendency to exhibit fear regarding risk-taking. Intellectually gifted students also demonstrate the need to reflect on their own thinking and learning (self-reflection); the need to achieve an understanding and an acceptance of the nature of giftedness; a need to develop healthy and positive skills for social interactions and relationships; and a need to solve problems in real life as a way to find meaning and connections in life.

Give an example of curriculum units with differentiated instruction for "academically diverse" students, explaining some of their beneficial characteristics. Explain how some researchers hope to inform the field of gifted education.

Some research groups interested in determining which methods of identification, testing, curriculum, and instruction are most effective for gifted students develop curriculum units to use in their studies. They then compare test scores of students taught using different curricula. Some researcher-developed curriculum units feature differentiated instruction in order to tailor teaching to the individual needs of "academically diverse students." These units place emphasis on problem-solving skills, conceptual thinking, and "real-world disciplinary inquiry" (e.g. using the scientific method to investigate subjects in various fields applied to real life). Curriculum units also function as tools for assessing individual students' learning needs. In addition, they help students attain progressively higher levels of expertise. Researchers believe that their work will foster both fairness and achievement in school systems nationwide and will nurture student gifts by focusing on gifted identification, model-based math and reading curriculum units, and traditional and performance-based assessments.

Describe some research questions related to curricula used in gifted education and based on theoretical models.

Some research teams, including teams on a nationwide level, are interested in examining comparisons of different criteria for identifying gifted students, the effects of such criteria on assessment results and on under-represented populations, and comparisons of the effects of general education curricula and model-based curricula on gifted students. They ask research questions such as: When comparing standardized and performance-based measures of reading and math, whether the criteria for identifying giftedness interact in any ways with the type of instructional method (i.e. general education curricula vs. theoretical model-based curricula) delivered. They want to discover if students not identified as gifted are helped or harmed in achievement by model-based curricula, and if non-identified students in their non-treatment and treatment classes perform the same or differently on standardized reading and math achievement tests. These researchers are also interested in how teacher implementations of model-based curricula are affected by personal and environmental variables.

Discuss expert opinions on creativity in gifted students as a function of cognition or personality. Identify some personality traits that researchers have associated with creativity.

Some researchers view creativity as a strictly cognitive process, others view it as a group of personality traits. Kerr and Gagliardi (2009) assert that while some students might be identified as creative in their thinking, without personality characteristics such as independence and persistence, these students may not produce any creative work. Hence they recommend that both aspects be considered. Divergent thinking is the most prevalent cognitive attribute studied in creativity research. With respect to personality, creativity has been most associated with the traits of autonomy, introversion, and openness to experience. (The latter two traits are included in the "Big Five" personality traits identified by McCrae & Costa and others.) Researchers have noted (e.g. King and Pope 1999, Feist 1999) that creative types are likely to work independently on their creative products and to avoid much social stimulation and group influence. Openness to experience affords the stimulation of new ideas, questions, and topics.

Describe some examples of provisions made by state Departments of Education for gifted students to accelerate in social studies from intermediate to high school classes and credit.

Some state Departments of Education (New York, for example) allow gifted eight-grade students to accelerate in social studies for high school diploma credit if the superintendent or designee ascertains the student's readiness. Eighth-grade students can be awarded this credit by the high school if they attend the high school with high-school students and pass the course with the same requirements as the high-school students. Alternatively, an eighth-grade student can gain high-school credit by passing the state proficiency or Regents examination, and the credit is accepted as a transfer credit by any high school in the same state. If the state has no proficiency examination or similar assessment in social studies, an eighth-grade student can pass a course in his or her middle/junior high/intermediate school that is approved for high school credit by that district's superintendent or designee. Local examinations indicating school principal-determined high school-level performance may also be used.

Explain how some gifted students' motivation for particular subjects may be at odds with their actual talents by giving an example. Discuss the implications of this disparity for student progress.

Some students are gifted in certain areas, yet their interests lay elsewhere, where they are less talented and even exhibit weaknesses. For example, a parent and retired educator had a gifted child with talent in mathematics and playing violin. However, his passions were for soccer and singing, where he was not gifted or average, but comparatively weak. His parents did not press him to focus on his gifted areas or give up on what he loved because he did not excel. Despite shortcomings in favored fields, his passion and persistence eventually won him a place on his high school soccer team and he realized his goal of making All-State choir in senior year. He teaches and directs voice and coaches soccer as an adult, happy with his choices. This illustrates that gifted students need not confine their efforts to gifted areas and may succeed in other fields with passion and perseverance.

Define and explain the terms mean, median, and mode as used in quantitative research designs, giving examples relevant to gifted education.

Statistically, mean, median, and mode are measures of central tendency, meaning they quantify trends wherein the majority of values fall near the center/middle. The mean/arithmetic mean is an average of all values obtained. For example, if in a group of 11 gifted students, four score 100% on a test, three score 96%, two score 94%, one scores 92%, and one scores 90%, the mean = 96%. If one students scored 70%, this would corrupt or skew the mean. The median represents the midpoint of all scores. In this example, with this distribution: 100+100+100+100+96+96+96+94+94+92+90, the midpoint is 96. Median does not inform the other values; if one student scored 70%, the median would not change. The mode is the most frequent value; in this example, 100. Mode does not reflect non-central, extreme values. If this example had four scores of 100% and four of 96%, it would have a bimodal distribution.

Contrast recognition and reinforcement of students gifted in music and athletics with that of students gifted in math and science.

Student musicians perform in concerts and receive applause from crowds of schoolmates and parents. Student athletes compete in games or meets, also attended by cheering crowds. In addition to internal motivations to use their talents, these students obtain powerful external reinforcement from this positive public attention. In contrast, students with mathematical and scientific gifts do not enjoy the same opportunities to show their skills regularly in such public arenas. Students gifted in academic subjects do not experience much formal recognition for their abilities outside of good school grades and teacher approval. However, such students also need to receive positive reinforcement in order for them to appreciate the great value of their gifts. They may also need to receive validation for their accomplishments, as some students may attribute their successes to their natural gifts and mistakenly feel that they have not added any effort to developing them.

Give some examples of motivational tools teachers can use to enhance math- and science-gifted students' appreciation of their own abilities.

Students gifted in certain fields such as music and sports, enjoy positive reinforcement received for performing in public arenas like musical concerts and athletic events where large groups of people, many of whom they know as fellow students and parents, applaud and cheer them. Students with gifts in math and science do not have the same opportunities for official public recognition. Educator Ed Zaccaro (2008) recommends "Einstein Awards." He designated very difficult math/science problems as "Einstein problems." When students solved one, he presented them in class with an award featuring a picture of Einstein. He reported dramatic responses from students: One parent told Zaccaro that his daughter said the award was the best thing that had ever happened to her. Another said she had to restrict her son to two hours of math per night as he always wanted to solve more Einstein problems. Simple positive reinforcement is a powerful motivator.

Explain how teachers can facilitate the enriched social studies reading and learning of gifted students by directing their attention to questions and during-reading strategies.

Students learn more from their reading if teachers instruct them to use what they read to answer questions. This focuses their attention on the reading and helps them apply it. Teachers should encourage advanced readers to think of their own questions and try to answer them through their reading. Many gifted students will already have formed their own questions out of their curiosity about the subject. For gifted students who are younger and/or not as proficient in reading, teachers also can create their own questions and use them as an outline for reading. This will help students to focus their reading to identify key points in the text. Teachers can write these question outlines to focus on the subject content or on student reading and study skills. A teacher also can identify during-reading strategies specific to the textual material to help students self-monitor their comprehension.

Contrast the expense of gifted pull-out programs versus inclusive, regular classroom-based gifted instruction by explaining some of the logistics of each.

Suppose that a school district has two schools served by a gifted resource teacher. A total of 60 gifted students are served, with 30 gifted students in each school. Each school has two classrooms per grade, and the gifted students are in second through fifth grades. Four grades x two classrooms = eight, x two schools = 16. With pull-out programs, the resource teacher can teach one group of students per grade level, four groups at each school (eight total). The teacher can do this in two full school days, one at each school. In contrast, if this teacher had to go to sixteen separate regular classrooms to teach the gifted students, it would take more time, and that time would be more fragmented. The teacher likely could not see students as regularly. Researchers found when a school district switched from pull-out to in-class programs, it cost three times as much to hire three times as many specialists as they had needed with pull-out teaching.

Provide examples of how teachers can integrate the arts into science curriculum for gifted students.

Teachers can give their gifted students exercises in applying particular scientific principles or concepts to solving problems in creating art. For example, they can assign students to explore the scientific subject of light. The teacher can assemble a group of paintings notable in their treatments of light (e.g. Rembrandt, Monet, Turner, and others) and ask students to discuss the manner in which each artist depicts light's interaction with color and with water, the directions of light in the paintings, the quality of indoor and outdoor light, the quality of outdoor light at different times of day, how this affects colors, etc. The teacher also can ask gifted students to choose some aspect of light of scientific interest to them and then to participate as artists themselves by coming up with ways in which they can express this aspect of light visually via sketches, drawings, paintings, collages, sculptures, etc.

Give examples of how teachers of gifted students can integrate arts goals and activities into their social studies curriculum.

Teachers can help their gifted students to associate the arts with social studies and vice versa by helping students relate such things as art movements with their social and historical contexts and events. For example, a teacher can ask gifted students to imagine they are time-traveling news journalists who visit the past to report o artistic movements such as Cubism or Expressionism. The teacher can assign the students to write newspaper articles telling how the historical, political, intellectual/philosophical, and social milieu and circumstances of the period influenced a movement in art. Students also can write about how the art movement influenced the thinking of the period in turn. They can explain how the art movement reflected the times and changes occurring, how the movement demonstrated a break from previous traditions, and how this break reflected and influenced historical and social developments.

Give some examples of ways teachers can nurture their students' areas of giftedness.

Teachers can observe student behaviors in order to identify their personal interests and abilities. When they notice a student excelling in a certain subject area, teachers can provide enrichment activities for that student. Even in a general education classroom with no gifted programs, teachers can provide additional activities in preferred areas to students who show evidence of giftedness by asking for additional activities. Teachers can vary their instruction by offering activities and assignments at higher grade levels to gifted students not challenged or even bored at their class' grade level. Teachers pressed for time can arrange peer tutoring by older and/or more advanced students to allow younger gifted students more 1:1 interaction and individualized learning at the student's own pace. Teachers can help gifted students plan and develop special projects in their areas of interest, allowing them to learn about these areas in more depth and with more complexity.

Provide examples of how teachers can integrate goals and activities in the arts into their writing curriculum for gifted students.

Teachers can use paintings, photos, and other visual art as catalysts to stimulate their gifted students to generate novel story ideas to write. They can suggest that the students use the visuals to correspond with their story's climax, its rising action just before the climax, or its denouement (falling action). Teachers can give gifted students practice doing investigative research by first giving them a story to read about a painting that vanished, and then having them imagine and write about how they, as "art detectives," recovered it. Teachers can assign students to write fictional pieces about uncovering art forgeries, including the telltale details. To give them experience with understanding various perspectives, teachers can have gifted students write historical fiction about the origins of famous artworks and points along their journeys to their eventual/current destinations. They can write from the viewpoint of the artist, people in possession of the art, people searching or it, or the artwork itself.

Discuss some teaching techniques to help gifted students in social studies classes understand an author's main idea by predicting it.

Teachers of enriched social studies to gifted students should realize the various demands that textual material makes of the reader. One such demand is to understand the writer's main idea. Teachers can help students prepare for such understanding by instructing them to anticipate the central concept in the text. Before they begin reading, the teacher can ask students to skim the text first to develop an educated guess regarding the main idea. The teacher should point out to students such clues as the title of the book, chapter, article, or essay; paragraph headings; terms and/or names that are repeated often; etc. The teacher and class should review all of the students' predictions of the key message or concept. They can revisit these predictions after students have read the text. Then the students can identify which clues they gleaned from skimming the text were useful and which clues were not.

Identify and summarize three sources or methods whereby gifted students can participate in an independent study program.

Teachers should ensure that any independent study program for a gifted student will meet their individual instructional needs as well as their particular personal interests. In self-selected studies, the gifted student chooses a topic she/he wants to study that will meet these needs and interests. In related studies or "spin-off" studies, the gifted student is acquainted with a subject in the usual teacher-directed class lessons, and then selects a topic within that subject that particularly interests her/him to investigate in more depth and specificity through an independent study project. In curricular studies, the teacher will have made a list of topics based on the subject content for the whole class to study. Gifted students can select one topic from this teacher-made list and conduct an independent study of it. Curricular topics may be studied independently by individual gifted students; or they may also collaborate on these in small group settings.

Explain how thematic units are used educationally and how they can be developed in virtual learning environments for gifted students. Give two examples of online resources to aid this development.

Teachers use thematic units as a way of incorporating complex, abstract concepts into the curriculum. Thematic units are organized around a particular theme or topic This organization provides a focus for learning abstract ideas and appeals to areas of students interest. One way that a teacher can employ both thematic units and information technology is to select the content for an integrated thematic unit and then develop a virtual learning environment (VLE) for teaching it. Today there are many Internet websites containing teacher resources, including those for thematic units. For example, the site A-Z Teacher Stuff offers thematic units such as magnets, oceans, Martin Luther King Jr., Harry Potter, and many other themes, including lesson plans for teachers, class activities, etc. The website of the C.O.O.R. Intermediate School District in Michigan provides additional examples of integrated thematic units.

Describe which subscales of the Wechsler Intelligence Scales for Children, 4th edition (WISC-IV, 2004) are good, fair, poor, and poorest measures of General Intelligence (g), and some additional considerations.

The Arithmetic subscale of the WISC-IV is found the strongest indicator of general intelligence (g); (Keith et al, 2004) and the strongest index of giftedness (Silverman, Gilman, and Falk, 2004). Vocabulary, Information, and Similarities, in that order, were identified by Keith et al as the next best indices of g. "Fair" measures of g were the Matrix Reasoning, Block Design, Word Reasoning, Comprehension, Letter-Number Sequencing, Picture Completion, Picture Concepts, Symbol Search, and Digit Span subscales, in that order. They found the Coding subscale "poor", and the Cancellation subscale the "poorest" measure of g. While Keith et al ranked Letter-Number Sequencing higher than Digit Span in their factor loading on g. Silverman and colleagues note that student responses to Digit Span are more predictable and interpretable, whereas Letter-Number Sequencing can seriously confuse and slow down some gifted students. They sometimes substitute Arithmetic for Letter-Number Sequencign with non-math-phobic-students.

Give a brief description of the Gifted Rating Scales (GRS) (Pfeiffer and Jarosowich, 2003) as a multidimensional screening instrument for identifying gifted children, including its age-level forms, included domains, and highlights.

The GRS are norm-referenced, based on multidimensional model of giftedness, and informed by current giftedness theories as well as federal and state definitions and guidelines. 1.The GRS-P form, for ages 4-6 years, 11 months, includes domains of intellect, academic readiness, motivation, creativity, and artistic talent. 2. The GRS-S form, for ages 6:0 through 13:11 and grades 1-8, includes domains of intellect, academic leadership, motivation, creativity, and artistic talent. Administration takes 5-10 minutes. Relative strengths and specific gifted areas can be identified. Statistical validity studies positively correlate the GRS with the Wechsler Preschool and Primary Scales of Intelligence (WPPSI-III), Wechsler Intelligence Scales for Children (WISC-IV), and Wechsler Individual Achievement Test (WIAT-II). These standardized scales are designed to help qualify students for placement in gifted/talented programs. Domain -specific identification guidelines are included. Teacher form completion is easy and quick.

Relate some details about what the tests in the Guildford Battery measure, how they are scored, and the status of research into their validity.

The Guilford Battery contains ten tests of different types of divergent thinking, a cognitive process associated with creativity. Half of the the tests are verbal measures, and half are nonverbal/figural measures. The verbal measures portion of the battery asks students to offer as many solutions as they can in the following categories: name for stories, verbal classes/categories, synonyms, sentence structures, verbal inferences about categories, The nonverbal/figural portion of the battery ask students to identify figures, categories of figures, systems of figures, transformations of figures, and inferences about the figures. Each test is timed and scored for fluency by number of responses and for originality by statistical infrequency of responses. Not much validity research exists regarding these tests. Follow-up studies by Meeker (1978) found that students identified as creative by Guilford's tests in elementary school also had high creativity scores in high school. Michael and Bachelor (1990) found limited confirmation of Guilford's findings through conducting factor analyses.

Identify and describe the Javits Act, explain its chronological relation to special education laws, quote its definition of giftedness, and illustrate how this definition incorporates special educational considerations.

The Javits Act, passed in 1988, after Public Law 94-142, aka the Education for All Handicapped Children Act (EHA) passed in 1975, renamed the Individuals with Disabilities Act (IDEA) in 1997, most recently reauthorized in 2004, with final regulations released in 2006. IDEA provides for all children to be educated, but it does not specify services for the gifted as for the disabled. The Javits Act, initiated by then - Senator Jacob Javits of New York, funds educational programs for gifted children from low-income families. It states, "The term gifted and talented student means children and youths who give evidence of higher performance capability in such areas as intellectual, creative, artistic, or leadership capacity, or in specific academic fields, and who require services or activities not ordinarily provided by the schools in order to develop such capabilities fully." This legislation meets the need for special services to gifted students in order to fulfill their special abilities.

Give a summary of the Multi-Dimensional Screening Device (MDSD) for gifted children, including its conceptual basis, a few steps in its implementation, and its ten categories of giftedness.

The MDSD (Kranz, 1978) is based on the idea that intelligence is multidimensional in nature, not reflected by any one measurement. The author designed this instrument to facilitate better initial identification of gifted ness among "the less accepted school population." A staff development program for teachers required to participate in the screening process is part of this instrument. In fact, the first step in its implementation is staff development. Other steps include rating students individually and forming a local screening committee to select individual students for screening. The MDSD's categories of giftedness are: 1. visual arts ability 2. performing arts ability 3. demonstrated creative/productive thinking 4. discipline-specific academic ability 5. general intellectual ability of 1 in 100 or more 6. leadership characteristics, organization and decision-making 7. psychomotor history and ability 8. history and use of spatial and abstract thought 9. wide difference between performance and general intellectual ability 10. talent related to cultural heritage

Describe some of the advantages of the online version of the Naglieri Nonverbal Ability Test, 2nd edition (NNAT-2) as an instrument for identifying gifted and talented students.

The NNAT-2 offers both pencil-and-paper and online administration options. The online version saves teachers and administrators a great deal of time compared with the time required for the pencil-and-paper version. It also saves money, as schools do not have to pay for printed paper tests. Furthermore, this test saves time, space, money and human energy by relieving schools of the need for storing, distributing, organizing, and shipping test booklets. It has minimal system requirements, enabling more users and computers to access it. The test's publisher, Pearson, uses a secure browser, protecting the safety of student information. Paperless, this test offers a more environmentally friendly option than does the traditional paper-and-pencil test. The online version yields instant result, allowing teachers to spend more time with students requiring additional, more focused instruction. Online reporting allows teachers to tailor data disaggregation and filtering to their district's needs. The online format is appealing and motivating to students, promoting test completion for fuller measurement. One item per screen helps students focus.

Describe an example of how research aims to clarify effective practices in gifted education.

The National Research Center on Gifted and Talented (NRC/GT) is seeking to find out "what works in gifted education" through a five-year plan beginning with the 2008-2009 school year. Their plan involves integrating the examination of systems of identification, reading and math curricula based on theoretical models, and assessments of giftedness. Researchers are concentrating on three ares: . developing a solid system of identification that will build upon prior research; 2. with students identified via both traditional criteria and expanded criteria, analyzing the effects of certain units of reading and math curricula; and 3. measuring student outcomes using standardized achievement tests, extended standards-based assessments, or structured performance assessments. Researchers aim to create a system of identification for students across socioeconomic and cultural groups. Such groups will feature "talent pools" of elementary school students, identified using standardized tests and teacher ratings, participating in randomly assigned reading and math curricula based on models.

Describe some aesthetic characteristics of intellectually gifted children in terms of the intensity of their sensory experiences, and provide examples.

The Polish psychiatrist and psychologist Kasimierz Dabowski (1902-1980) proposed intensities he termed "overexcitabilities" or "supersensitivities" in gifted children. His concept of sensual intensity involve heightened awareness and perception of the senses of vision, hearing, touch, smell, and taste. Gifted toddlers with this sensitivity may hate the feel of grass on their bare feet, gifted children may feel nauseated at the smells of certain foods, and/or they may overeat other foods that taste especially good to them. This characteristic is consistent with a craving for pleasure and a need or desire for comfort, two additional characteristics of gifted children with sensory sensitivities. These children often are more sensitive to air pollution and/or to the discomfort of labels/tags in clothing. Another aspect of this sensitivity involves the aesthetic appreciation of beautiful jewelry, visual art, music, literature, and the beauties of nature. Children with this sensitivity often can be moved to tears by beauty.

Summarize the purposes of the section of the Gifted IEP (GIEP) for Specially Designed Instruction (SDI). Summarize some considerations relative to general education teachers/programs and GIEPs.

The Specially Designed Instruction (SDI) section of the GIEP is, in general, the section where classroom activities are designed to match the individual gifted student's needs. Any specific educational challenges or issues for that student, including any problems the student has in the general education classroom, should be addressed here. Any educational programming that the student and his/her parents think should become part of the school district's gifted program should also be included in the SDI section. Three considerations relative to general education teachers and GIEPs are: 1. gifted education and general education for gifted students are not separate entities; gifted students' needs must be met in BOTH district gifted programs AND general education classrooms 2. general education teachers cannot refuse to implement GIEPs; parents have recourse (i.e. another GIEP meeting/due process) if teachers refuse to implement 3. general education teachers have access to all parts of GIEPs

Describe some disadvantages of the Wechsler Preschool and Primary Scales of Intelligence (WPPSI-III) as an assessment instrument for young gifted children, and Provide some alternatives for addressing these.

The WPPSI-III is a standardized instrument with proven validity and reliability. However, it does have some disadvantages. In testing the youngest children, the WPPSI-III's subscales reflect an expectation for IQ scores to jump significantly every 2-3 months. To qualify children for gifted services, experts advise scheduling testing optimally for ruling in/out such qualification. They also warn that a retest can take up to two years, so if a child has taken the WPPSI-III and needs re-evaluation sooner, they should use a different test. Another consideration about the WPPSI related to the youngest children is that IQ results are not as stable at earlier ages. Therefore, administering the WPPSI-III near the end of the kindergarten year is recommended to allow greater maturation and pre-academic/academic skills development. An exception is when earlier testing is indicated, e/g. a child displays signs of extreme giftedness at a very young age.

Name the age range, subscales, and available composite scores of the Wechsler Preschool and Primary Scales of Intelligence (WPPSI-III. Describe some of its features, and some considerations for using it and for choosing between the WPPSI-III and WISC-IV (Wechsler Intelligence Scales for Children) when applicable.

The WPPSI-III is for ages 2.6-7.3 years. It includes 14 subscales: 1. block design 2. information 3. matrix reasoning 4. vocabulary 5. picture concepts 6. symbol search 7. word reasoning 8. coding 9. comprehension 10. picture completion 11. similarities 12. receptive vocabulary 13. object assembly 14. picture naming By various combinations of subtests, measures can be computed for verbal IQ, performance (fluid) IQ, processing speed quotient, general language composite, and full-scale IQ. Verbal IQ performance, performance IQ, and full-scale IQ are computed from core subtests. The WPPSI-III features short activities to accommodate attention spans of the youngest children in the age range. Activities are colorful and game-like, appealing to and interesting young children. Varied samples, second chances, and layered scoring enabling partial credit help children do their best. With children aged 6.0-7.3, WISC-IV for ages 6:0-16:11, is recommended for identifying giftedness, the WPPSI-III for general academic reasons. Because they are similar, these should not administered successively to avoid practice effects. (choose one or the other-not both.)

Define affective process models used in gifted education. Identify one such model and its components.

The affective domain relates to the emotions, emotional responses to the environment, and interactions with the social environment, including an individual's motivation, attitudes, perceptions, and values. Some process models used in gifted education focus on this domain. Krathwohl's Taxonomy of the Affective Domain is one such model. It contains five levels: 1 - Receiving: Students show awareness of and attention to stimuli presented by teachers. 2 - Responding: Students commit to discover, seek out learning activities, and feel satisfaction from participating in the process. 3 - Valuing: Students demonstrate decision-making about a value, their engagement with it, and their commitment to it. At this level, students may endeavor to convince others to commit to a value they have chosen. 4 - Organizing: Students construct a belief system/value system, including attitudes, which they organize by the interrelationships of its components. 5 - Characterizing by a value or set: Students have organized and internalized their value system and can apply it o many circumstances as philosophy of life.

Describe the collaborative consultative model of education relative to the instruction of gifted students.

The collaborative consultive model of education posits three component entities: the Consultant, the Mediator (the consultee), and the Target (usually the student). This model views the consultant and consultee as equal partners. They bring varying expertise to the collaboration and have mutual responsibility for identifying problems, planning strategies for interventions, and carrying out recommendations through combining their respective knowledge and skills. Disagreements between the consultant and consultee are regarded as constructive opportunities to extract the most helpful information. This model assumes that consultant and consultee will both work directly with the target or student. Educational researchers have found in surveying teachers that while time shortages and scheduling issues can interfere with the success of collaborative consultative services, most teachers find them acceptable as alternatives to resource room services; furthermore, some teachers find them very desirable as alternatives. This model addresses special students needs, including giftedness as well as disabilities.

Discuss how many common characteristics of gifted children can influence their affective development differentiate from that of non-gifted children and the implication for educators.

The developmental tasks for gifted children are the same as for all children. For example, all children must develop such concepts as a sense of identity, a sense of self-efficacy, differentiation of self, relationships with peers, autonomy in life and in school, and paths toward careers. But gifted children can experience these developmental milestones quite differently from other children. Researchers find gifted children are commonly more emotionally sensitive, intense, and overly excitable. They also are commonly more perceptive about human behaviors and personalities; along with their talents, their moral development is precocious, so they tend to have different concerns at higher cognitive levels and younger ages than other children. Even the faster cognitive processing of gifted youngsters can intensify their emotional responses to external stimuli. These common gifted characteristics may sometimes impede developmental processes. Therefore educators, parents, counselors, psychologists, and psychiatrists working with gifted students should possess adequate and accurate knowledge regarding these children's affective development.

Generally describe the process whereby a gifted student should conduct and independent study project, including how parts of this process benefit the student's intellectual development.

The gifted student chooses the topic of special interest and read extensively about it, developing a knowledgebase. With a general grasp of the topic and knowing what he or she wants to discover, the student asked questions to answer. Students utilize and develop higher order cognitive skills as they learn to identify and use resources in the forms of places in persons, establish goals and objectives's, developing carry out plans to accomplish these, make self evaluations of their own work, and identify avenues whereby they can communicate their discoveries, conclusions, and knowledge. Students can use fact cards to record information they collect on the topic. By analyzing the information they gather, students can develop questions reflecting a problem to solve; for example, the student can ask what methods she or he could use to get a specific species of wildlife off the endangered list. This process demands student analysis of information, creation of novel and unusual plans, and expression of their opinions. It also involves them in authentic research, products, and audiences.

Discuss some environmental factors that have positive and negative influences on the expression of creativity in gifted individuals.

The influence of environmental variables can be seen historically through creative proliferation at certain times and places. For example, Florence, Italy in the 15th century; the Harlem Renaissance in 1920s and 1930s New York in America; and San Francisco, California in 1960s America all exhibited unusual amounts of creative inspiration and production. Some factors that allow people with creative gifts to express them in meaningful and socially valuable ways include: social support for a subculture consisting of creative persons; availability and accessibility of materials and resources for creative production; a political and cultural climate that allows freedom of expression; and the existence of patrons with the means and interest to lend financial, material, and moral support to creative endeavors. Influences impeding creative expression include the race, gender, and socioeconomic status of some gifted individuals due to others' stereotyping and lowered expectations because of such environmental characteristics.

State the main purpose and nature of the gifted IEP (GIEP) relative to the nature of the gifted student's learning style and needs. Summarize the steps to take if a GIEP team has modified a student's GIEP to make it more appropriate and effective but the student is still experiencing educational problems.

The main purpose of the GIEP is to address the needs of the individual gifted student. By definition, a gifted student learns differently enough from others that standard teaching methods and curricula are unsuitable. The different nature of the gifted student's learning and needs is the reason for a GIEP. The nature of the GIEP is not a class(es) or program(s) in which the gifted student participates. Instead, the GIEP offers a plan for the student's education. As such, the GIEP must be monitored: the student's educational needs will evolve/change over time, and the plan then must be adapted to continue meeting them. If problems persist after modifying a GIEP, the team should revisit the student's Present Levels of Educational Performance, ascertain what additional assessment data and other information are needed, and develop a new, more suitable/effective GIEP. Teams should obtain outside expert consultations if needed.

Characterize the general attitudes of most educators toward academic acceleration for gifted students and those of educators with specialized experience and expertise in giftedness. State some factors contraindicating and easing acceleration.

The majority of research studies show that academic acceleration for gifted students affords them many benefits in both academic and social domains. However, despite these positive findings, most educators are reluctant to adopt or even consider acceleration as an option for fear that acceleration will result in problems in the students' emotional and social development. In contrast, educators who specialize in the education of the gifted student, and teachers and parents who have had personal experience with the acceleration of gifted students tend to have much more positive attitudes about accelerating and are more willing to select this option when a gifted student's profile indicates it. Pushing a student to accelerate can cause problems, as can accelerating an emotionally immature student or one continually getting negative responses from peers or educators. Having a small support group of similarly accelerated students eases the acceleration process.

Give and example of how mismatches between learning styles and teaching styles can contribute to underachievement in gifted students who are members of minority groups. Card 2

The majority of teaching methods in public schools tend to use the verbal modality, to deal in abstract concepts, and to teach concepts an skills in isolation, i.e. taken out of context. These methods contrast markedly with the learning styles of many minority students. For example, research finds that black students are more likely to be visual rather than verbal learners, i.e. they learn better using visual images than words. In addition, black students are more likely to be field-dependent rather than field-independent learners, meaning that they rely on the surrounding context to understand the material presented and will not grasp it as well (or at all) when it is presented out of context. Finally, the research indicates that black students tend to learn more concretely rather than abstractly, so teaching abstract concepts through using concrete objects rather than presenting such concepts only in abstract terms would enhance their learning. Mismatched teaching methods produce confusion and frustration in students with these learning styles, thereby leading to underachievement.

Express one of the most common concerns of parents and educators about acceleration regarding the gifted child's wellbeing. Explain how research findings address this concern.

The most common worry of parents and educators about accelerating a gifted student to a higher grade is the impact it might have on the student's emotional and social development and wellbeing. They fear that while the child's intellectual development in undoubtedly at the higher grade level, his or her social and emotional development might lack the same levels of advancement. They worry about the child interacting with older children. However, researchers find children who are emotionally well-adjusted and socially comfortable before acceleration will report : one from their newer class of older students, and another from their previous age-level class and/or other children their age, with whom they maintain friendships. While gifted children who have trouble making friends may experience difficulty in older classes, many gifted students prefer friendships with older children, finding more in common with them due to their advanced development in many areas.

Define what instructional accommodations a student, parent, teacher, or advocate can request of a school district for a gifted student, including some common responses and their validity.

The only constraints on accommodations one might request in school for a gifted student are that such accommodations be reasonably expected to provide the student with meaningful educational value using the school's curriculum. Any accommodations meeting these criteria can be written into a student's gifted IEP. A common response from school/district administrators for students, parents, and advocates to consider is that no one ever has asked for a certain accommodation before, or no others are requesting a particular change. This happens more frequently in schools using only pull-out gifted services and/or those schools not normally providing accommodations to general education classrooms. Administrative responses even can give students/parents/advocates the impression that their request is inconvenient and/or unnecessary. However, rejecting a request simply because it sets a precedent is not valid. The only criteria for validity are the reasonable expectation of meaningful educational value and its educational appropriateness for the individual student.

Give an example of how mismatches between learning styles and teaching styles can contribute t underachievement in gifted students who are members of minority groups.

The predominant cultural values in America favor individualism, competition, and excellence in individual achievement, both in society and in school. In contrast, many other countries have cultures that value collectivism, cooperation, and placing the good of the group ahead of one's individual concerns. Therefore, many students belonging to minority groups in the United States have cultural values that more closely reflect the values of other countries of origin than the majority of American values. These students tend to be more social and cooperative in their attitudes and behaviors, and less individualistic and competitive. Teaching styles in America tend to place emphasis on competing with others to be the best. Too much emphasis on competition can interfere with minority gifted students' achievement: rather than increasing their motivation, competition can decrease motivation by provoking anxiety and by negatively influencing students' social and academic self-images.

State the primary value of self-contained classes for gifted students and explain one curricular and instructional consideration that impinges on this value. Identify one pitfall for general education classroom teachers when all gifted students are in self-contained classrooms.

The primary value of self-contained classes for gifted students is the academic rigor possible when all students in a class are at similar intellectual levels. The general classroom teacher will not have to differentiate instruction specifically for gifted students by offering more in-depth study of subject content, materials at higher grade levels for acceleration, compacted curriculum, alternative activities, etc., if no gifted students are present. However, one inherent pitfall is that some general education teachers, relieved of the necessity to differentiate instruction for gifted students, may fail to differentiate instruction for their existing students. Such failure reflects that these teachers do not understand (or observe) that differentiation is intended to customize teaching for the needs of every individual students, not just every individual gifted student. Varying difficulty levels, paces, prior knowledge, interests, and learning styles also require differentiation among non-gifted students.

Give a general description of a program placement option for gifted students consisting of regular education with resource service.

The term "resource services" refers to the student in the general education classroom going to a separate resource room part-time, also called a pull-out program or send-out classes, available in some middle schools and more prevalent in elementary schools. Typically, groups of 10-30 students attend a resource room for classes with a gifted teacher for one day or 1/2 day weekly. Gifted teachers may reserve one weekday for third-graders, and designate two other days for cross-grade groups of fourth- and fifth- and/or fifth- and sixth-graders in the resource room. They might visit regular classrooms to teach model lessons on a fourth weekday, and use the fifth day for completing their administrative duties.

identify instructional models that provide a basis for reading curriculum units used in researching effectiveness in gifted education. Give one example of how research studies select sample schools to participate and how they measure outcomes.

Three instructional models are: 1 - Carol Ann Tomlinson's Differentiation of Instruction Model, which focuses on varying teaching methods 2 - Sandra N. Kaplan's Depth and Complexity model, which focuses on attributes of deeper understanding and more complex knowledge; and 3 - Joseph S. Renzulli and Sally M. Reis' Schoolwide Enrichment Model (SEM), which uses educational enhancements that simultaneously benefit all students and address the special needs of gifted students. The National Research Center on Gifted and Talented (NRC/GT) has a longitudinal study basing curricula on these instructional models. By sampling school districts nationwide that implement gifted and talented programs, these studies' findings represent diverse populations, socioeconomic levels, and urban, suburban, and rural settings. These sample schools are selected from nominations by educators who develop curriculum models; state gifted education directors; and representatives from state and national organizations. Outcomes are measured by (1) state standards extended for advanced reading and math achievement; and (2) performance-based measures of investigative and problem-solving skills.

Identify and define three types of qualitative research used to study giftedness and/or gifted education.

Three types of qualitative research used to study giftedness and/or gifted education are... 1 Case Studies - This method seeks to gain insights into a phenomenon such as giftedness by studying one individual case of the phenomenon in great depth and detail. The case could be an individual gifted students, a specific class of gifted students, a school for gifted students, or a particular event involving gifted students. 2 Phenomenology - refers to the process wherein the researcher describes the structures of his or her experiences as they are apprehended by the researcher's consciousness, without using theories, assumptions taken from other fields/disciplines, or deductive reasoning to influence that description. 3 Ethnography - the researcher makes detailed field observations of sociocultural events or characteristics, typically within a particular community or population. This type of research can lend itself to studying groups of gifted students and gifted educational programs.

Discuss what elements of oral communication should be taught to gifted students in school programs for teaching the language arts.

To attain a good balance, language arts instruction in the area of oral communication for gifted students should address equally both speaking and listening to oral language. Gifted students should be assisted in developing their skills for evaluative and critical listening - not just hearing, comprehending, and remembering spoken language, but also analyzing, assessing, and judging it for veracity, accuracy, credibility, and relative bias, and distinguishing between facts and opinions. Gifted students should learn and practice the techniques of formal argumentation and debating. In addition, gifted students should practice discussion as another element of oral communication, including asking questions, probing for additional information, and expressing and hearing ideas and then building upon these collaboratively. Students talented in the creative arts will benefit from the opportunity to develop advanced skills in these areas through instruction in oral interpretation and participation in dramatic production.

Articulate five principles related to a high quality of gifted classroom instruction.

To be of sufficiently high quality, classroom instruction for gifted students should accomplish the following: 1 - It should adapt, modify, or differentiate the grade-level curriculum and teaching practices to address the special educational needs of gifted students; 2 - It should give gifted students ways in which they can show their proficiency in the required curriculum, and thereafter give them educational opportunities that present them with appropriate challenges; 3 - It should be made up of curriculum options, resource materials, and teaching approaches that are differentiated and reside along a continuum; 4 - It should allow for flexibility in educational arrangements for gifted students, to include such options as the use of compacted material for accelerating or advanced students; subject and/or whole-grade acceleration; independent study agreements and research projects; and 5 - It should be designed with the goal of increasing the depth and breadth of knowledge that learners with high ability acquire.

Provide examples of how teachers can integrate goals and activities in the arts into their writing curriculum for gifted students. Card 2

To develop gifted students' skills for synthesizing information, teachers can have them use a variety of sources such as paintings, photographs, musical compositions, and literature or other written work, to write an essay, script, poem, or sketch about some current event they find reported in a newspaper. The teacher can assign students to write an account of this event from various points of view, such as their best friend; their mother; their father; their sister; their brother; their family's pet dog, cat, or other animal; their teacher; etc. This helps students explore both the variety of sources for news stories and the variety of perspectives afforded by different individuals, as well as helping them to synthesize disparate elements. Another writing exercise integrating the arts involves having students listen to and analyze a musical composition and then write a script for a conversation analogous to the music in tone, tempo (speed/pace), divisions, and changes in speed, pitch, etc.

Describe some ways that schools can remediate some of the drawbacks of pull-out/resource room programs for gifted students.

To remedy lack of coordination between pull-out activities and regular classroom curriculum, the resource teacher can circulate a newsletter to classroom teachers explaining their resource room activities and how they connect with the regular curriculum. To provide both gifted resource teachers and general classroom teachers with reciprocal insights into each other's work and experiences, they can each trade places for a day or half-day. The gifted resource teacher can offer to teach a lesson for the regular classroom teacher, who can observe the teaching strategies the resource teacher uses for gifted as well as regular students. The gifted resource teacher also can propose that the two teachers work together to team-teach a regular class that includes gifted students. Educational researchers find that pull-out programs can succeed or fail depending on whether or not the regular and gifted teachers share mutual understanding, solid communication, and positive interactions.

Explain the rationale for differentiating instruction in the language arts for verbally gifted students in K-12 education.

Traditionally, instruction in the language arts has placed emphasis on basic reading skills and on assessing those skills by testing lower-level cognitive skills such as factual knowledge rather than fostering active inquiry and learning. Gifted students who have already mastered fundamental reading skills will not be challenged by these traditional methods. Verbally gifted children frequently attain linguistic proficiency at younger ages than their non-gifted age peers. Individual gifted students may be more advanced in a variety of areas in addition to reading, such as literary analysis, writing poetry, and/or writing prose. Gifted students who have mastered basic reading skills are ready before others to apply their abilities to higher-level cognitive tasks such as reading critically, writing exposition, communicating orally with others, developing their vocabularies and language usage, and learning foreign languages. These differences dictate a need for differentiated instruction for verbally gifted students in all developmental stages.

Discuss some differences between intelligence and creativity measures for indicating giftedness.

Traditionally, standardized intelligence tests were used to determine whether a student was gifted. IQ scores above certain numbers (e.g. above 120, 130, etc.) qualified a student as gifted. However, IQ scores are an indication of intellectual giftedness rather than of creativity. Traditional standardized intelligence tests typically feature questions or problems with only one right answer. When taking this kind of intelligence test, the student must use convergent thinking to narrow down the possibilities to one correct one. In contrast, many measures of creativity test divergent thinking, which generates multiple ideas and possible responses. During the 1960s, J.P. Guilford and E.P. Torrance both developed standardize tests of divergent thinking that are still popular today. Because the traditional standardized intelligence tests require convergent thinking and little or no divergent thinking, and because divergent thinking is identified as a key characteristic of creativity, scientists have come to hypothesize that intelligence and creativity are separate constructs.

Relate some considerations expressed by experts in the field for gifted students in public schools and their parents related to Gifted IEPs

Two main concepts regarding gifted education (as expressed by McIntyre and Mery, 2004) are 1. educationally, gifted does not mean that a student necessarily demonstrates superior academic performance in the regular, general education classroom. Rather, it means that the student has an educational need. The gifted child learns differently enough from others that regular classroom, grade-level teaching methods and practices are insufficient. 2. Once this educational need is identified, the school must plan for the student's individual education to meet his or her specific needs. This planning is reflected in the development of an IEP. In addition to the need to individualize the student's educational planning, the authors emphasize that gifted "pull out" programs alone are insufficient, as students still spend around 95% of their time in regular classes. Pull-outs should be used together with modifications to the regular program. Also, accommodations/services must not be limited to group programs, precluding individualization.

Identify salient characteristics of low-income black parents whose gifted children have achieved well in school, according to research findings.

Various researchers have found that in low-income black families, the parents' amount of education did not influence their gifted children's achievement; rather, parents' attitudes and behavior exerted the greatest influence on children's achievement. The parents of high-achieving students in this population assertively involved themselves in their children's education and kept track of the children's progress. These parents perceived themselves as competent in their coping skills and expectations for their gifted children. They supported the ideology of achievement, and their orientations toward achievement were positive. They established specific, clear, achievement-oriented norms for their children. They also created explicit and clear boundaries regarding roles. They purposefully practiced behaviors and provided experiences to promote their children's achievement. These parents' relationships with their gifted children were openly communicative, supportive, nurturing, trusting, respectful, and positive.

Describe the benefits of learning other languages, some methods of teaching foreign languages advantageous for gifted students, and some choices of languages.

Verbally gifted students cannot only excel in English, but also should learn foreign languages. In fact, foreign language study is one of the areas of curriculum differentiation in language arts instruction for verbally gifted students. Learning other languages enhances understanding of the history and structure of English; exposes students to the histories and cultures of other countries; gives them access to literary, scientific, and other text in foreign languages; and enables proficient students to communicate with persons from other nations and backgrounds. Such instruction even can prepare some students for future diplomatic careers. Learning foreign languages early is advantageous for gifted students. They can and should accelerate through four years of a second language and at least two years of a third. Choosing languages spoken in the community will afford follow-up opportunities. Spanish, French, German, Chinese, and Japanese are good choices. Latin and Greek (also a modern spoken language) are invaluable sources of English word roots.

Give some examples of actions teachers should avoid that might interfere with gifted children's development.

When a child exhibits giftedness in certain areas, the teacher should not prohibit or curtail the child's interest in preferred subjects and activities. If a gifted child prefers some subjects to the exclusion of other required curriculum, the teacher can arrange behavioral contracts or contingencies; e.g., completing an assignment in a non-preferred area can be rewarded with more time exploring a preferred interest. Teachers should not punish or disapprove extensive interest in certain subjects, a characteristic of gifted students. Teachers should not force gifted students to follow traditional procedures for learning and performing if they learn best with different procedures. As a student, Albert Einstein reputedly was failed in math by a teacher for not following the prescribed procedures, even though he got the right answers. Fortunately, he overcame this deterrent through his intellect, curiosity, and persistence. Teachers should not ignore gifted students who grow bored or who clamor for more material.

Give an example of how a teacher can integrate art and math in curriculum with an activity for gifted students that enhances both the ability of visual perception and the ability to estimate mathematical quantities.

When creating visual, especially representational art, artists frequently self-assess their visual perceptions against reality by estimating objective distances, sizes, heights, widths, and areas in the physical world. Teachers can provide their gifted students with comparable experiences through real-world experiments. For example, they can go outside on a sunny day in an area with some trees, each push a stick into the ground, measure the length that is above ground and then measure the length of the stick's shadow. Then they can measure the length of a nearby tree's shadow. The teacher then asks them how they can calculate the height of the tree, in inches and in feet, based on the information they have. Teachers should allow gifted students to come up with their own methods for solving the problem, including drawing diagrams, taking photographs, or whatever they want to try. This activity sharpens accuracy in both visual perception and numerical estimation.

Cite the percentages of values in a statistically normal distribution that fall within 1,2,and 3 standard deviations from the mean. Discuss the implications of this distribution of values for gifted education using a simple hypothetical numerical example.

When data from a group can be graphed as a bell curve, they have normal distribution. The larges number of scores will fall on or around the mean or average. In normal distributions, approximately 68% of the values fall somewhere within 1 standard deviation above or below the mean; about 95% of the values fall within 2 standard deviations above or below the mean; and about 99% fall within 3 standard deviations above or below the mean. A steep curve with most values near the mean shows a small standard deviation; flatter curves show larger standard deviations with more values spread away from the mean. If the average IQ score in a regular classroom is 100, with one gifted students having a score of 150, that score represents 1% of the population, as it is more than 3 standard deviations (typically 15 points with IQ tests, x3=45) above 100.

Explain the Intrinsic Motivation Principle of Creativity and how it has been developed and tested.

While what Renzulli has termed "schoolhouse giftedness" is more easily identified via IQ test scores and more amenable to academic success than what he called "creative-productive giftedness," which is not as easily identified via IQ test scores. Social psychologists conducting research into creative production over 25 years have found it associated with students' motivational orientations, and those orientations are influenced by environmental factors. They find that internal motivation encourages creativity, while external motivation discourages creativity-this finding is called the Intrinsic Motivation Principle of Creativity. Intrinsically motivated students do tasks for their own sake because they enjoy them, so their rewards come from within them. In contrast, extrinsically motivated students do tasks for a reward coming from outside the.

Identify and define four main parts of a Gifted IEP and their purposes.

When developing a Gifted IEP (or any IEP in general), the first part should indicate the student's Present Levels of Educational Performance (PLEP). Because asynchronous development is characteristic of gifted students, they have relative strengths and weaknesses as do other students, and they also may be extremely gifted in specific areas. Therefore the PLEP must reflect school performance in all school subjects. This establishes a baseline wherefrom subsequent educational programming proceeds. The second IEP part should state the student's educational goals. Goals are more general and global; they do not include specific ways of achieving them. A third IEP part includes short-term learning objectives. These objectives do specify the time frames in which to achieve them and exactly what measures will demonstrate the learning. These are steps in achieving goals. A fourth part outlines specially designed instruction for the gifted student and explains teaching methods for attaining short-term outcomes and long-term goals.

Identify some of the special needs of gifted students teachers should consider when selecting curriculum materials.

When teaching gifted students, teachers will find that while these students often master textbook content easily, they also demand broader and more in-depth knowledge of a subject than textbooks afford. As they see additional curriculum materials for gifted students, teachers should consider their special needs. Specifically, students with higher levels of ability need to be presented with more sophisticated ideas than others and more challenging activities than those usually offered in general education classrooms. Choosing activities that can be individually tailored to any specific student will give teachers more options for optimally meeting individual gifted students' educational requirements. Curriculum materials chosen for gifted students should require more use of higher-order cognitive skills than for other students. Teachers should select materials that incorporate relevant major themes, contain high levels of abstraction, allow student exploration according to their interests, and enable them to create products.

Describe how teachers of enriched social studies can help gifted students with during-reading strategies related to the Five W's of journalism, comparisons and contrasts, and cause-and-effect relationships.

When their social studies students are reading journalistic content such as newspaper or magazine articles, teachers, should include in their assignments the identification of the "Five W's" of journalism: Who, What, When, Where, and Why. Answering these questions helps students focus on the key elements of persons, actions, times, places, and reasons in historical or current events. Teachers should call students' attention to comparison/contrast, showing how the author saw similarities and/or differences between/among events, actions, or situations, and why the author found these similarities/differences important. Teachers can show how authors describe cause-and-effect relationships with qualifying phrases like "as one result...", "in part because of...", "this helps to explain...", etc., and then have students compile lists of such qualifiers. Teachers can help students explain authors' causal arguments without agreeing with them and differentiate opinion from fact by asking students how the author explains the causes of an event, rather than asking them what the causes are.

Identify the roles of America's federal and state governments in defining giftedness. Summarize the manner in which American states address this definition.

While America has federal laws governing special education services for students with disabilities, these laws do not contain specific provisions that mandate each state to offer special education services for gifted and talented students. As a result, each American state can create its own definitions of giftedness and talent and its own programs for gifted and talented students. A state's definition will inform how it identifies such students; how it decides eligibility for services; and the kinds of educational programs it develops. Indiana, Nebraska, and Washington use the term "high ability students." Eighteen states use "gifted," while the other twenty-five states use "gifted and talented." Four states-Massachusetts, Minnesota, New Hampshire, and South Dakota-have no state definition of giftedness or talent. Giftedness and talent are defined by the state legislatures in 25 states, while the remaining 21 states have each mandated or authorized their state Boards of Education to define these terms.

Explain how procedural safeguards are established in government relative to how gifted students are identified, assessed, and found eligible for services, and provide a few examples.

While federal law provides that students with designated disabilities are guaranteed a free appropriate b\public education (FAPE), it does not address gifted students. Thus each individual determines the procedural safeguards for gifted students' identification, assessment, and eligibility for services, just as each state also determine its definition of giftedness and gifted programs. Some examples of procedural safeguards include 1. requiring school districts to give parents prior written notice before identifying, evaluating, placing, changing, or providing a FAPE to their child 2. obtaining written parental informed consent before initially evaluating their child to determine eligibility and before initially providing services for gifted children 3. parents' right to examine their child's school records and participate in educational planning meetings for their child 4. parents' right to due process hearings to resolve disputes related to identification, evaluation, or placement of their child or provision of a FAPE

Identify the purpose, uses, and six main components of the Multiple Menu Model of curriculum development (Renzulli et al, 2000).

While its authors originally conceived of the Multiple Menu Model of curriculum development as a means for differentiating instruction to address the needs of gifted and talented students, Renzulli remarks that teachers can also use it to promote original inquiry and creativity in students at all levels. This model uses the term "menu" to indicate that teachers are able to choose from many options within each menu. The six component menus are: 1 - Knowledge Menu - addressing the specific subject area 2 - Instructional Objectives and Student Activities Menu 3- Instructional Strategies Menu 4 - Instructional Sequences Menu 5 - Artistic Modification Menu - menus 2-5 address various aspects of pedagogy, i.e. instructional techniques 6 - Instructional Products Menu (consists of the interrelated Concrete Products and Abstract Products menus) and addresses the kinds of products the student may create based on the knowledge acquired, and how students as primary inquirers construct this knowledge.

Describe some desirable features of differentiated language arts curriculum and learning for gifted students in the area of literature.

While standard school literature courses afford a number of works of high quality for students to read, gifted students will be expected and will want to go beyond these. High school students can find reading lists of books for pre-college preparation at most public and school libraries. Some gifted students read at such a gigh level that some of these books may be accessible and suitable for them in middle/junior high or even elementary school. Recommendations from educators include that teachers should emphasize student development of critical reading and thinking skills and help gifted students to develop their skills for analyzing and interpreting the literature. Teachers are also advised that their gifted students should read a broad range of subject matter. They should become familiar with various authors and their contents, themes, and styles. They should also come to recognize which authors are their favorites and learn more about those authors' lives.

Identify several factors contributing to a prevailing view of student gifts and talents as assets with "non-asset" emotional and social aspects being addressed less.

While superior intellectual abilities and/or creative talents may seem obvious assets, a number of factors have contributed to this view's overshadowing the recognition of less beneficial aspects giftedness. Accordingly, samples selected for quantitative research studies have not covered a wide enough range of the gifted population. By not including sufficient socioeconomic, cultural, and performance factors in sampling, studies do not reflect the true range of issues for gifted students. In addition, such research finding inform the development of instruments for assessment and intervention, which in turn will not adequately represent "non-asset" aspects of giftedness. Also, due to the paucity of qualitative studies on gifted students, researchers are limited in understanding feelings and thoughts that gifted students have not expressed to others. Other factors include our society's emphasis on equality, and legislation mandating educational accountability and achievement, both of which are not conducive to addressing gifted students' emotional and social needs.

Identify the roles of US federal and state governments regarding procedural safeguards are related to the educational placement of gifted students. Identify some procedural safeguards are coming across individual states.

While the federal government has mandated procedural say guards for the educational placement of students with disabilities, federal laws do not specify the clean address the placement of students who are gifted without disabilities. Each state department of education issues at One procedural safeguards for the placement of gifted students. They typically derive the standards from the federal requirements for students with disabilities in that all students must receive a free, appropriate public education (FAPE). Some common safeguards across states include the following standards: district must give parents prior notice of initiation or refusal to identify, evaluate, place, or provide a FAPE to a gifted student, in the parents primary communicative mode. Paris must be given copies of procedural said guards. Written informed parent consent is required before providing individual formal valuation or initial service to get to student, and such consent is voluntary and revocable before the action. Parents have the right to view their child's educational records and participate in educational planning meetings.

Relate some potential issues to consider when applying the collaborative consultative model to gifted education in schools.

While there are many definite advantages for using the collaborative consultative model to meet students' special needs, including those of gifted students, there are also a few potential issues to consider. First, teachers, parents, and other participants from outside the school may have little or not training collaboration, unless the school institutes such training. If the school does so, another consideration is the time, expense, and human energy involved in providing training. Another very real consideration is that any or all of the potential collaborators may not have sufficient time to interact with one another. In addition, some educators prefer working with students, and would rather not work with other adults, especially if the lack relevant experiences. Moreover, implementation of collaborative consultative methods requires firm support from the school administrator. Finally, while collaborative consultation is effective, visible results take time, so patience is needed.

Describe a few features of each of the following types of differentiated instruction for gifted students: acceleration, complexity, depth, challenge, and creativity.

With acceleration, the master educational standard has fewer tasks assigned. Students are assessed prior to instruction. Acceleration clusters are determined by higher-order determined by higher-order cognitive skills. With complexity, multiple higher-level thinking skills are used. Use of multiple resources is required. More variables are added to study. With depth, students study multiple applications of a concept they have learned. They might conduct original research into subjects they study, or they may develop an original product. With challenge, students make use of advanced resource materials or work with more sophisticated content in subjects they study. They can apply their learning across different academic disciplines. Furthermore, they are expected to be able to explain their process of reasoning to others. With creativity, students might design and/or build a model based upon principles and/or criteria they have learned. Alternatives for various academic tasks, products, and assessments are made available. Oral and written communication to the real-world audience is emphasized.

Explain an activity using "language experience stories" wherein you can help a younger gifted child develop language skills in a way that is interactive and fun.

Younger gifted children with advanced abilities in language often will enjoy activities using language experience stories. You ask the child to make up a story on any subject the child prefers. For example, the story can be about the child's pet; the child's best friend or circle of friends; a favorite relative; a trip they took on vacation that they especially enjoyed; a day trip to a museum, zoo, planetarium; a party they had or attended; or any other subject of the child's choice. They child tells the story aloud and you write it down. You have the child draw illustrations to accompany the story. You then read the story back to the child, aloud from the printed version. Record your reading. Play back the recording and listen together, having the child follow the printed words to associate spoken and written language. Make multiple stories into a book for the child.


Related study sets

Life Insurance - Section 1 - General Insurance

View Set

Strategic Management: Chapter 13

View Set

CHAPTER 48 Management of Patients with Intestinal and Rectal Disorders QUIZ

View Set

Chapter 12, Section 4 (Vocabulary Terms + Study Questions)

View Set

Routers, Switches, Bridges, Hubs

View Set

MODULE 10: HEALTH, AGING, & THE ELDERLY

View Set